You are on page 1of 79
GMP-1516-PH-ASSIGN-174 ‘TEST YOURSELF BUBIECTIVE PROBLEMS EF of resistance In the figure shown there is a conducting loop ABC per unit length 2. placed near by a long straight current carrying wits, ‘The dimensions are shown in the figure. The long wire lies in the t plane of the loop. The current in the long wire varies as I= Tot. Find (a) the mutual inductance of the pair (b) the heat produced in the loop in time t. A thin non conducting horizontal disc of mass m having total charge q Bekt distributed uniformly over its surface, can rotate freely about its own axis. Initially when the disc is stationary a magnetic field B directed perpendicular to the plane is switched on at t = 0. Find the angular velocity @ acquired by disc as a function of time, if B = kt, where t is time. A charged point particle having positive charge q, mass m is projected in x-z plane with a speed vo at an angle 0 with the x- Eo axis, while another identical particle is _ projected t simultaneously along the y-axis from the origin (see figure) yo & with an equal speed Vo. ae ‘An electric field E=~E,k exists in the x - z plane, while a = : magnetic field B = B,k exists in the region y > 0 (Eo, Bo > 0). It is observed that both the particles collide after some time. Assume that the space is gravity free. (i). Find the time after which the particles collide and the coordinates of the point of collision. (ii) Find the velocity vo and the angle 0 in terms of the other variables. (iii) What is the maximum value of the z-coordinate of the first particle? Ina YDSE experiment the two slits are covered with a transparent membrane of negligible thickness which allows light to pass through it but does not allow water. A glass slab of thickness t = 0.41 mm and refractive index }4=1.5 is placed infront of one of the slits as shown in the figure. The separation between the slits is d = 0.30 mm. ‘The entire space to the left of the slits is filled with water of refractive index p1y=4/3 con te find A coherent light of intensity I and absolute wavelength 4 = 5000A° is being incident making an angle 30° with horizontal, If screen is placed at a distance D = 1 m from thesis, (a) the position of central maxima, (b) the intensity at point O. PMIIGE Lid., FUTJEE House, 29-A, Kals Sarai, Sarvapriya Vinar, New ‘website: wins fi In the Young's double slit fut, the medium between sitet refraction index varying with times asa ont (a) find the y co-ordinates of central my @) Its velocity as a function of time, (0) Ifa glass plate of small thickness p j front of S How should its refractive: raced in with time so that central maxima ig < ae vay ato, ‘A gas containing hydrogen like ions with atomi: = Z, The n= Z, These photons fall on a metallic plate and gj wavelength 1 of SA°. Find the value of °Z’ eject electrons ha ‘own in the Seren me maxima In the nuclear reaction p+ °N—> $X4n (a) Find A,Z and identify the nucleus xX. (b) Find the Q-value of the reaction, (©) If the proton were to collide with the initiate the above reaction. @ Ifthe proton has the twice th i withthe diction ofthe incident proton, ie aentmot erase m(p) = 1.007825u — m(SC) = 15.0106 u m(SN) = vitae m(SN) = 15.000u - m('S0)= 15,9949 u i m(n) = 1.008665u (0) = 15,0031 u and 1 u~ 931.5 MeV 15 'N at rest, find the minimum K.E. needed by the proton to ‘An electron is orbiting in a circular orbit of radius r under the influence of a constant strong magnetic field of strength B. Assuming that Bohr’s postulate regarding the quantisation of angular momentum holds good for this electron, find (@) the allowed values of the radius ‘r’ of the orbit. (0) the kinetic energy of the electron in orbit (© the potential energy of interaction between the magnetic moment ofthe orbital current due to the electron moving in its orbit and the magnetic field, B. (@ the total energy of the allowed energy levels (©) the total magnetic flux due to the magnetic field B passing through the nth orbit (Assume that the charge on the electron is © ‘and the mass ofthe electron ism.) A photoelectric plate is initially exposed toa spectrum of hydrogen gas excited to second ot level. Later when the same photoelectric plate is exposed t0 @ spectrum of some unknown Iyrogen ke gs cacited to second encray level, This found thaiiie de-Broglie wavelength of the photoelectrons no ejected has increased ‘V6.1 times. For this new gas difference of energies of first Vea os Balmer series limit is found to be two times the ionization energy ae atom is gr ji te, Detect the atom and determine the work function of the photoelectric plat rex lens using the UV method the following ‘values In the measurement of the focal length of a conv @em A0-em a 30.5 em 33.cm ea in wand v are casurements. I the errors Cae sore A peal Yengih in atypical measurement ; tof fo 0.4.om respectively, estimate the error 1" the ment cae ee GMP-1516-PH-ASSIGN-176 1 A cylinder of mass m rests in a supporting block as shown. If f = 60° and @ = 30%, calculate the maximum acceleration a which the block may be given up the incline so that the cylinder does not lose contact at B. (neglect friction anywhere) 12. A block of mass m is attached to the frame by a light spring of stiffness k. The frame and block are initially at rest with x = Xo, the uncompressed length of the spring, If the frame is given a constant horizontal acceleration a towards left, determine the maximum velocity (VreiJmx of the block relative to the frame (block is free to move inside frame). Ignore any friction. 13, A heavy disc with radius R is rolling down hanging on two non-stretched string wound around the disc very tightly. The free ends of the string are attached to a fixed horizontal support. The strings are always tensed during the motion. At some instant, the angular velocity of the disc is @, and the angle between the strings is o.. Find the velocity of centre of mass of the disc at this moment. 14. A small particle of mass m is attached at B to a hoop of mass m and radius r, whole system is placed on the rough horizontal ground. The system is released c from rest when B is directly above A and rolls without slipping, Find the angular acceleration of the system at the instant when AB becomes horizontal as shown in the figure, 15. A uniform rod of mass m and length L is placed on the fixed cylindrical surface of radius R at an small angular position 6 from the vertical (vertical means line joining centre and vertex of the cylindrical path) as shown in the figure and released from rest. Find the angular velocity © of the rod at the instant when it crosses the horizontal position (Assume that when rod comes at horizontal position its mid point and vertex of the circular surface coincide). Friction is sufficient to prevent any slipping, 16. A small ball of mass 100 g is attached to a light and inextensible string of length 50 cm. The string is tied to a support O mass m released from point A which is at a horizontal dis 30 cm from the support. Calculate the speed of the ball at it point of the trajectory. FINES Lid., FUTIEE House, 29-A, Kalu Sarai, Sarvapriya Vihar, New Delhi 1 - = (eee 2. 2, GMP-1516-pH., As toy car is moving on a closed track SIGH-177 A acent graph describes the variation K whose curved portions are i speed o otis point F), The time t required for the cart compte the car with distance moved to (2a Im. The 2 One lap is equal 0 6k second Fra natn om Find k. (take mind = 2 Ren Bn den Denil Ave atm) A particle P is to be projected from a fixed point A on the ground fixed speed u. Another particle Q is also to be projected cael a B which is directly above the point A. The trajectory of the particle Q touches all possible trajectories (for different angle of projection) of the particle P in the vertical x-y plane. Find the equation of trajectory of the particle Q. (Take the point A as origin, horizontal axis x-axis and vertical axis y-axis. Consider the figure.) ‘A small coin is placed on a stationary horizontal disc at a distance r from its vertical centre, The disc starts rotating about a fixed vertical axis through its centre i with a constant angular acceleration a. Determine the number of revolutions N, accomplished by the disc before the coin starts slipping on the disc. Find the coefficient of static friction between the coin and the disc is}. ; : ee ea - ‘A wedge of mass M resting on a horizontal frictionless surface . tipon by a force F in the horizontal diction as show figure. Findie § ——- 7 van upon by a toa force acting on the shaded portion of the weds ive i along the A person rolls a small ball with speed u alo oor from: point A. fy xed cele a required speed u so that the ball retums i is ical plane rolling on the circular surface in the vertical a vertical from B to C and becoming 2 projectile at c. z fem of (Neglect friction and radius of ball) i i dis at point B. A very small i js in relaxed state its free e foe i pee i is smoot! . Det where, eat ‘of kinetic friction isle ae . 5 a wales sow it ae. Compression 8 so that the block enters Comic FRURG Lid, FITJEE House, 2A, GMP-1516-PH-ASSIGN-178 23. 24, 25. 26. 27. rough surface The wheel of radius R rolls without slipping on horizontal rough surface, and its centre O has an horizontal acceleration ap in forward direction. A point P on the wheel is a distance r from O and angular position 0 from horizontal. For a given values of ap, R and r, determine the angle © for which point P has no acceleration in this position. A.car C of mass mis initially at rest on the boat A of mass M tied to the identical boat B of same, mass M through a massless inextensible string as shown in the figure. The car accelerates from rest to velocity vo with respect to boat A in time ty sec. At time t= to the car applies brake and comes to rest relative to boat in negligible time. Neglect friction between boat and water find the velocity of boat A just after applying brake. ‘An adiabatic piston of mass m equally divides an insulated t container of total volume yo and length ¢. A light spring connect [Pe | the piston to right wall. The container has helium gas, The ae 2, pressure on both side of piston is Py. The container starts moving ee with acceleration ‘a’ towards right, find the stretch of the spring eZ when acceleration of the piston equals acceleration of container. (Assume displacement of the piston << é) For the arrangement shown, a cylinder of mass m with cross- 4 sectional area A, initially in equilibrium position, is displaced slightly inside the liquid of density p. Prove that the motion is simple harmonic and also find its time-period. A block of mass m = 1kg is attached to a free end of a m, spring whose one end is fixed with a wall as shown in the figure, The block is performing simple harmonic & Sagem (0) es motion. The position of the block from O is given by x= 24 Jsin2t, where xis in meer and tis in second A particle of same mass is released fom the circular path ata height h = 80 om. then particle it and sticks to the block. The collision takes place when velocity of the block is zero and S88 elongated, Find (i). Time period of new SHM. (ii) Maximum velocity of the system 29. 31 3 34, 35, a er sonvaprva SMP STH, ‘A closed organ pipe of length ¢ taba is 5® harmonic mode with tod Gea in at two points £ and 3¢ from one end, Ifthe feng of Wo, A SARK ‘= rod is 4¢ and it is vibrating in first overtone a t SSesdse ‘ Find the length of the rod. [Velocity of sound i i in air ‘Two strings of the same length but differen suspend a rod whose density varies as p figure, The frequency of standing waves pri ratio n : 1, when both the strings are vibrating images et’ Sting is in the the mass per unit length of string (1) is we net fundamental mode, If of the other string. 1 (1) is Ho calculate the mass per unit length Mina ‘Oung’s modulus for the rod Y and der ‘unit length are used to“ — Po [1 + ox} as shown in the Mt Mass per In the figure shown, an electric dipole an infinitely long rod of linear ae ey eee (@) Find the net force acting on the dipole? (©) Whats the work done in rotating the dipole throx i ig the dipole (© If the dipole is slightly rotated about its Spllisoa pace : find the time period of oscillatic is tea eo lation. Assume that the dipole is | Sia A charge q is placed at a height = from the base of a cone of height h and ] h radius R as shown in figure. Calculate the flux linked wit mie lux linked with the curved surface Two parallel large metal plates, each of area A are placed at a distance d apart. Light of intensity I and wavelength 4 falls on the plate 1 which is qd photosensitive. The efficiency of photoelectron emission is 100%, The —> photoelectrons are immediately swept away from plate 1 in such a way that > the plate 2 remains neutral. Find the potential difference between the two _ plates at any time t, if plate 1 is neutral initially. Find the equivalent resistance between A & B in the given network. Pins capacil i i shown, C2 is Two identical tors are connected in the circuit 5 bias ‘0 100 V and switch S is shifted from position 1 to position ps ‘The negative terminal of C2 is connected with the positive a a ict C,. Find ive (ere done by the batiery. If the polany © 5 reversed, what work would have been done by the In the figure shown the reading of voltmeters are © V, = 40V, V2 = 40V and V3= 10'V- Find Lave (@) the peak value of current (©) the peak value of emf (©) the value of L and C. a (@) the value of C for w ch the circuit will ian Qrieoneni) produce maximum heat WMNNKE Lid, FITUEE House, 2A, Kale pst: wisntiee co GMP-1516-PH-ASSIGN-180 36, 37, 38. ) 39. 20 20 sh of the given ida charge on each ba 10 ie Find the current in each brand circuit in steady state, Find the capacitor. [fie yyy to 30 Gag onnected across a resistor Rat t= 0. The separaigg A capacitor initially given a charge Qo is : (st<1). A small bulb is connected across the 4 between the plates changes according to d= ae d : plates ofthe capacitor which lights when potential difference across the plates ofthe capacitor reachg ‘Vo. Find (a) the variation of charge with time (b) time when the bulb will light. ‘The resistance each of 16 © and capacitance of each 100 pF are arranged as shown in the figure. A battery of emf 12 Vis joined across A and B. Find the (@ reading of the ammeter just after key is closed and after long time, (ii) charges in each capacitors when steady state is achieved. Two infinite parallel wires, having the cross sectional area ‘a’ and resistivity ‘k’ are connected at a junction point ‘P” (as shown in the figure). A slide wire of negligible resistance and having mass ‘m’ and length ‘¢” can slide between the parallel wires, without any frictional resistance, Ifthe system of wires is” (oo introduced to a magnetic field of intensity “B’ (into the plane of paper) and the slide wire is pulled with a force which varies with the velocity of the slide wire as F = Foy, then find the velocity of the slide wire as a function of the distance travelled, (The slide wire is initially at origin and has a velocity v , ) ‘A conducting ring of mass m and radius r has a weightless conducting rod PQ of length 2r and resistance 2R attached to it along its diameter. It is pivoted at its centre C with its plane vertical, and two blocks of mass m and 2m are suspended by means of a light in extensible string passing over it as shown in the figure, The ri 5 free to rotate about C and the system is placed in a magnet; aie Gino the plane ofthe ing) A circit is now completed by oon : the ring at A and C toa battery ofemrV. Fad tis\yalee sears the system remains static, io PATUGE Lid., FUTUEE House, 29-4, Kalu Sarat ee 4 44 45. ome. MPS LPH ASSN. 19) pleotric field given by the vector (ity) we : t NIC is y plane. A small ring of mass M carrying char Metin Freely on a smooth non : . freely conducting rod. is projeteay oy NHiCh can slide (my the point (0, 6 Such that it can reach the siaeuats along the tod from there is no gravity in the region. What minimum Mee, fod. Assuming ity should be siven tothe ring? ifin $.1 unit 2 M 4 A point source of heat of power P is Placed at the ces ‘ material of = “et has thermal conductivity K. ifthe te a spherical shell of mean radius R. the the inner surface of the shell is not to exceed T, find the thickness of the shel ne ome om : sss of the shell In the figure, OA and OB are two uniformly wires, placed along x and y axis rae Find te unit vector in the direction of field at point (0, 0, ¥3L). In the laboratory, the high emf of a battery is measured by using potentiometer and two resistance Ry and Re, where Ry << R, as shown in the figure. Ry = (100 + 0.10)62 and Rz = (9900 + 9.90), AB = Im ‘The voltage across Ri, when switches S, & Sa are connected to point 1 and point 2 is balanced against 6 = (60 £0.06) cm. ‘when the switches S; and S2 are shifted to point 3 and point ¢ ts shown in the given figure, whe potential difference of standard Ey = 2 volt is balanced against length és = (15 © 0.075) cm. The maximum error in ‘emf of battery E is volts (upto two decimal places) and mass m is projected into a region of uniform electric field of strength E,, with velocity Vs Pet cular to E,. Throughout the motion apart from electric force particle also experiences @ dissipative foree ‘of constant magnitude qE, and directed opposite to its i Je of 90°. velocity, If | V, Fe cs i ped he es umn ner A ‘A charged particle having charge 4 i ‘Ww ius ‘r’ laced ont ‘A uniformly conducting wire is bent to form a os ‘m’ 2 nant = te eagle Ks a 3 og haizonl uae pce There gh de ring in avery small ime nies y Doras ie . Tete oh inna magnate eld of induction 8 ecomes vertical. Calculate a a 8 ee avin) in terms of ae given quantities. “Assume that friction 1S ignore any loss in energy. GMP-1516-PH-ASSIGN-182 48, 49, 50. st, ircle of radius R and a charge craven et the figure. Find the ratio wer it as shown i nitude of the electric field (En), A non-conducting wire is + Q is uniformly distributed 0 (%) of the potential (Va) to the mas diameter AB that bisects the wire (as oth evaluated at the other end A of shown in the figure) In the figure ABCDEFA is a closed conducting wire loop. The Sone Part AFE is a portion of an ellipse, aoa =land a BCD is a portion of the ellipse Ne St 9a? 4a? A uniform magnetic field =B,k exists in this region. If the frame is rotated about the y- axis with a constant angular velocity ‘o’. Find the maximum emf induced in the loop. In the figure B,, Bz and B, represent uniform time varying magnetic fields in three different infinitely long cylindrical regions with axis passing through P, Q and R and perpendicular to the plane of the paper. PQR is an equilateral triangle of side 4V3 r, where ris the radius of each cylindrical region (r= im), The magnetic inductions vary with time t as B, = A + at, By = A—bt and B, =A + ct, Here a= 2 Tesla/s, b = 2 Tesla/s and ¢ = 1 Tesla/s and A is a constant with dimensions of magnetic field and ‘t’ is time. Find the induced electric field strength at the centroid of the triangle PQR. Two blocks A and B of masses m and 2m respectively are connected together by a light spring of sites kand tea placed ona smooth horizontal sufice, The blacks are socked otsae cost such that spring gets compressed by a length xo and then released from rest. Find the work done on the block A by the spring, by the time the spring acquires its natural length. Consider the circuit shown in the figure. Capacitors A and B, each have capacitance C = 2 F. The plates of capacitor A are shorted using a wire of resistance R = 1 © while the left plate of capacitor B is given an initial charge Q = +4 C, The switch is closed at a time t = 0, What will be the initial current (in ampere) drawn from the battery immediatel switch is closed? ates a med Lid, PUTUEE Howse, 25, Rahs Sarat Setar Toe ne uate: une ore aw f, Given figure is the part of some circuit, Chay % ine circuit is given by q=3 (3-e*) Be on the capacitor in » Coulomb) where t is time in oom. Let Va. Ve and Vo be the potential : respectively. At t= itVas SV autres A, Band ¢, flows through R = 1Q, then find the V, (in oon io= 1 Amp f length 1 m rotates abo it 3, Arod o! : ut one of its end : velo 2 rade a plane perpendicular to the eee | eae shown in the figure. Then find magnitude of electric ficld (in SI unit) at “yh"If * * the mid point of the rod. (in ST unit) at “tm 4, A certain series RC circuit is formed usi i dilectic having a capacitance C=2F and abaley afenfE< SY Tiecieat fgg is completed and it is allowed to attain the steady state. After this, at t = 0, half the thickness of the capacitor is filled with a dielectric of constant K = 2 as shown in the figure, The system is again allowed to attain a steady state. What will be the heat generated (in joule) in the capacitor between t= 0 and t= co? —i— 55. Abeam of light having wavelength 5000 A” is incident on a plane metallic surface. The intensity of the incident beam of light is 40 watt and its cross-sectional area is 2mm?, What will be pressure (in dyne/ cm’) exerted by radiation on the metallic surface if 50% of it is reflected back by metallic surface. 56. Two soap bubbles of equal radii (R=8cm) are stuck together with an intermediate. film separating them. ‘Surface tension of solution forming bubbles is 7x10? Newtor/ meter, What will be distance between the central of soap bubbles (in cm), ‘stance r from a point source listen the sound emitted by the source. When 57. A person standing at dis int having distance a from the point source, then increase in sound. person move to another poi intensity level is nx10' decibels value of nis? io ee ee 8 Fi hows a smooth cylindrical pulley of radius R with centre at origin of ead is ideal roel is thrown over it on the two parts of eal thread two identical masses are tied initially at rest with co-ordinates eo » and (-R, -R) respectively. If mass at x-axis is given ah Ee i leaves contact with pulley at cos, Rsind). Then g/sind. 9. Object © is moving with a velocity 8 mis} a Th fd te ams moving with a velocity of 6 mV/s as sh Ce : velocity of image with respect to object 1100 rr New Deli 1100 Witiee La PHOEE House, 29-8, Kab S27ah SOE SE yun fier com —————=——— ~~ GMP-1516-PH-ASSIGN-184 60. 61, 62. 63. 65. 67. 68. ’) eu cas Find the value of magnetic dipole moment Nara C the following circuit the current distribution is shown in the &% gral | ' ita 2m in ic Hi illumi lic surface and ejects photoelectro ‘A monochromatic light source of frequency v illuminates a metal F < The photoelectrons ae ‘maximum energy are just able to ionize the: hydrogen atoms in ground state When the whole experiment is repeated with an incident radiation of frequency (Sv/6), the photoelectrons so emitted are able to excite the hydrogen atom beam which then emit a radiation of wavelength 1215 A®. Then the frequency v will be equal to k x 10° Hz. Then find the value of k A thermally insulated vessels contains two liquids with initial temperature qT and T> and specific heats Cy and 2Co, separated by a non conducting partition. The partition is removed, and the difference between the initial temperature of one of the liquids and the temperature T established in the vessel turns out to be equal to half the difference between the initial temperatures of the liquids. Determine the ratio of M,/M; of the masses of the liquids. Ina uniform gravitational field g, which is acting vertically downward, a ball is thrown at an angle with horizontal such that the initial (immediately after projection) radius of curvature is 8 times the minimum radius of curvature. If angle of projection is 156°, then find the value of 0. ‘A wall consists of alternating blocks with a length d and coefficient _d of thermal conductivity 4, = 2 W/mK and 42= 4 W/mK. The cross- sectional areas of the blocks are the same, Find the coefficient of | [— =] thermal conductivity of the wall (in W/mK). % h A ball of radius R = 100 cm is located in the way of propagation of a plane sound wave. The sonic wavelength is 2.= 2 cm, the frequency is v = 100x Hz, the pressure oscillation amplitude in air is (AP) = 4 Pa, Find the mean energy flow rate (in watt), averaged over an oscillation period, reaching the surface of the ball. Take density of air 1 kg/m’, Two small balls of the same size and of mass m; and m (m, > m3) are tied by a thin light thread and dropped from a large height. Determine the tension T of the thread during the flight after the motion the balls has become steady. (Air resistance force is same on two balls) [Take m, = 2kg, m= 1 kg, g= 10 m/s] AA pulsar is a rapidly rotating neutron star, the result of the gravitational collapse of an ordinary $ that has used up its fuel supply. A certain pulsar has a rotational period of T = 0.033 sec and = R = 1/38 km. The tangential speed of a point on the equator is (km/s) A family enters a winter vacation cabin whose walls are ‘the same as the outside temperature (0°C). The cabin consig and height 3 m. The room contains a 2 KW electric h and that all the heat from the heater is absorbed by after the heater is turned on, the air temperature rea find x (Take C, = 20.8 J/mol-K) GMP-1516-PH-ASSIGN-184 60, 61 62, 63. 64. 65. 66. 67. 68. FIG Lid, PUTIEE House, 25-A, Kaki Sarai, Sarvapriya Vihar, New Delhi-T Find the value of le moment (in Am’) of the 2m. he value of magnetic dipo ¢ r following circuit the current distribution is izta ia i ic surface and ejects photoelect ‘A monochromatic light source of frequency v illuminates a metallic I fines ‘The photoelectrons having maximum energy are just able to ionize the hydrogen atoms in ground state When the whole experiment is repeated with an incident radiation of frequency (Sv/6), the photoelectrons so emitted are able to excite the hydrogen atom beam which then emit a radiation of wavelength 1215 A°. Then the frequency v will be equal to k x 10! Hz. Then find the value of k. A thermally insulated vessels contains two liquids with initial temperature T; ‘and T and specific heats C, and 2Cp, separated by a non conducting partition. The partition is removed, and the difference between the initial temperature of one of the liquids and the temperature T established in the vessel turns out to be equal to half the difference between the initial temperatures of the liquids. Determine the ratio of Mi/Mb of the masses of the liquids. Ina uniform gravitational field g, which is acting vertically downward, a ball is thrown at an angle with horizontal such that the initial (immediately after projection) radius of curvature is 8 times the minimum radius of curvature, If angle of projection is 150°, then find the value of 8. ‘A wall consists of alternating blocks with a length d and coefficient © __d_, of thermal conductivity 41 = 2 W/mK and = 4 W/mK. The cross- sectional areas of the blocks are the same. Find the coefficient of thermal conductivity of the wall (in W/mK). h A ball of radius R = 100 cm is located in the way of propagation of a plane sound wave. The sonic wavelength is 2.= 2 cm, the frequency is v = 1007 Hz, the pressure oscillation amplitude in air is (AP)= = 4 Pa. Find the mean energy flow rate (in watt), averaged over an oscillation period, reaching the surface of the ball. Take density of air | kg/m’, Two small balls of the same size and of mass m, and im (m, > ms) are tied by a thin light thread and dropped from a large height. Determine the tension T of the thread during the flight after the motion of the balls has become steady. (Air resistance force is same on two balls) [Take m, = 2kg, m= 1 kg, g= 10 nv/s’) A pulsar is a rapidly rotating neutron star, the result of the gravitational collapse of an ordinary st that has used up its fuel supply. A certain pulsar has a rotational period of T 70.033 sec and ats R= 1/38 kin. The tangential speed of a point on the equator is (km/s) A family enters a winter vacation cabin whose walls are adiabatic, initially the interior temperate * the same as the outside temperature (0°C). The cabin consists of a single room of floor area 6 mby 4 and height 3 m. The room contains a 2 KW electric heater. : and that all the heat from the heater is absorbed by the air, after the heater is tured on, the air temperature reaches c find x (Take C, = 20.8 J/mol-K) website: www fitjee.com. BB. 74. B. 6. Water is filled in a uniform contai A. A hole of cross wectiot wane Of area of cross ge container at a height of 20 m above ie A) i made ty Ce out and hits a small block on surface nt ce, Water st ie container. Block is moved on surface in psa’, “stance ‘aon, always hits the block, The initial velocity of tera that stream e oof 'e block (ii is Given = 3) Take g = 10 ms) ae 600 J of heat is added to a monatomi ic Bas ina process i 150 J. The molar heat capacity for the process 2 KR. Then en 2 Pesos a work of n find K. A cavity is taken out from a unifo: ing sphere, Inside the cavity a dipole is placed a oH : Find the potential at point P (in Volt), Bees =10°C, £=0.1 = 309, d= G mum, 0 = 30°, d= 10 cm, R= 12.cm) ‘The minimum distance between an object and i Jength fis Kf. Find the value of K. object and its real image formed by a thin convex lens of focal In : Contes tube, the potential difference used to accelerate the electrons is increased from 12.4 kV to 24.8 KV. As a result, the difference between the wavelengths of K, -line and minimum wavelength becomes thrice. The wavelength of the K, line is 0.25 x KA® Find the value of K fe _a2.4kVA)) e ‘A particle P is initially at a distance d = 16 m from a fixed point O, The particle P moves with a velocity 7=SBO+31. Where BO is a unit vector from P 10 O at any time t Initially PO is perpendicular to 1. Find the time in seconds after which point P meets point ‘A-rod of length ¢ = 2m is maintained to rotate with a constant angular velocity « = 1 rad/s about vertical axis passing through one end (fig). @s There is a spring of spring constant k= 1 Nim which just encloses rod inside it in natural length. One end of the spring is ae axis of rotation. S is sleeve ‘of mass m= 1kg which can, just fit on. fr ‘All surfaces are smooth. With what minimum kinetic enerey Gs sleeve should be projected so that it enters on the rod without impulse and completely compresses the spring. iti i . A man AB is a platform of length 10 m kept Of frictionless horizontal ee ey of same mass is standing on end A. Friction coefficient betwee A Goeer el man and platform is 0.5. ‘Man starts running from A and om at B. Find the minimum time (in seconds) required by the person to arrive at B- e i spof mass m. A is connected to colg ‘i to partis ight inextensible thread AB. Initially line OB is vertical. System fa aH Deas ‘AandB are a, and az respectively. Find S xy 3 meee pan 110016 401 WNNGE Lid, FUTJEE House, 2A, Soe le ‘wun flys 0" GMP-1516-PH-ASSIGN-186, 78, 79. je 2 isa light pipe lying ina vertical plane and of cross sectional area 1 cm” whose small Aria areancned fix the a “An ideal fluid of density 10° Korat is being flown through the pipe by means of external agency. Length of BC a REESE The minimum vertical upward force required to pull out the pipe from grot ree om Ate s|, should be maximum velocity (in m/s) of the fluid so that pipe can not be pulled out from ground? Figure shows the VT diagram for helium gas in a cyclic process. Find the ratio of maximum and minimum pressure. v 2y| Tat Sane S; and S, are two equally intense coherent point sound sources vibrating RB in phase. The resultant intensity at P, is 80 SI units then resultant intensity : at P2 in SI units will be w/o an ~ omPn MPUSTEPH ASSIGN. 107 inthe circuit shown in the figure, th (a) with both switches open.” "* ‘al ammeter reading ¢ curren {b) with both switches closed eae ‘The readings are same in the c “ases, Th © Ne value of resistance R ig (A) 20. ©452 Pens (D)7.5Q A square plate of mass 10 kg and side 20 m is moving along the groove with the help of two ideal roller (massless), connected at the comers A and B of the square as shown in the figure, At the given instant of time during motion (as shown in the figure), the comer A is moving 16m with velocity 16 m/s downward, Find the speed of corner D( approximately), (A) 32.5 m/s (B) 16.5 m/s (©)8.5 mis (D) none Two geometrically identical homogeneous balls are manufactured of different materials. Their densities ball-1 and ball-2 are 800 kg/m’ and 200 kg/m’ respectively. Both balls are dropped from very high tower. Assume that the air resistance is proportional to the square of velocity. Determine the ratio of the maximum velocities of the ball ~ 1 to ball-2. (A)2 (@)3 (4 @)6 ‘A metal plate of arca 0.10 m? is connected to a 0,04 kg mass via a Siring that passes over an ideal pulley (considered massiess a frictionless) A liquid with a film of thickness of 0.3 mrt eo a between the plate and the table, When system is released, the Pith moves up with constant speed of 0.085 ns. Find the appiny ts 2 value of coefficient of viscosity of the liquid. (mass of metal pl 0.02 kg, g = 9.8 m/s” . ner ees 0 3461 104 Pase (C) 6,90 x 10° Pa sec @) ai A A cubical transparent slab is used as a paper weight, What should be the minimum refractive index o the material of the slab so that letters below it are not eh from any of its vertical faces. (A) v2 (D) ¥5N3 (v5 7” een 110016, (UGE Lad PITRE Nowe, 20, Kae ara, SAMPTYE iy ftjoe com. GMP-1516-PH-ASSIGN-188 6. is i 40 cm from a glass lens, close 10 the pring An object of size 2 cm is placed at a distance of n 5 : al any the Jens, and the lens forms an image of size 2 cm, What is the size of the image and hay a image is formed if the lens and the object, in the same arrangement, is immersed into ya . i Jens is 1.5, and that of the water is 1.33. — yal Sa ee at 4m in size. (B) virtual image, upright and 4 cm jn Size, (C) real image, upright and 2 cm in size. (D) virtual image, upright and 2 cm in sing WHA A thin and light thread of sufficient length L VY Yj is attached to a wire frame, The frame (along . Uj with the thread) is dipped into soap solution. GF Y- j, When the frame is taken out of the solution, 4 the thread takes the form of a semicircle with @ soap film extending between the frame and the thread as show in the figure (a). Now the thread is deformed into two semicircles by applying a force of F at the middle of the thread, as a result the film figure (b). Calculate the surface tension of the soap solution. fig. (a) fig. (b) Bets stretched as shown in ty Li aE (A) Be Tt Or 2nF LaF Oa (rise ‘An ice floe is floating in the sea. The part which is above the water is a prism of height 0.5m. A209. Xe seal climbs onto the floe, which sinks $ om deeper into the water. How many seals, which have he same mass, can the floe hold without sinking totally into the water? | (A) 10 (B)20 | (© 30 )40 A capillary tube is made of glass of refractive index n'. The outer radius of the tube is R. The tbe is filled with a liquid of refractive index n >d) has a M r uniform surface charge density such that the electric ficld just outside a the curved surface of the cylinder at point M is Eo, Find the electric “G20. field due to charge distribution at point P (>> £), z td fd ta (A) Ey > CE (A) E55 @rRa ORS 21 The space has electromagnetic field which is given as B having mass m and positive charge q is given velocity v,i at origin at t= 0 sec. ‘The z-coordinate of the particle when it passes through the z-axis is (neglect gravity through motion) 2n'mE, 4°mE, 0 8 (Cee (A) Bi @) Be © ee @) 22, An ideal monatomic gas undergoes a cyclic process ABCA as shown in the figure. Graph of pressure versus volume of the cyclic process ABCA is 35, ‘An infinite uniform current carying wire current Ip in the direction of the positive Satu G une Z-axis, canying sil (where all the points are equally spaced), wna OEP ferent i rnlre at point (4m, J B-ag—He ie 0m) and radius 4m as shown in the figure, If " i it SA unit, then the value of k is (A) 4 O3 D § ‘An uncharged capacitor C and a variable resistance Rare connected to an ideal source and two reaets A with the help of a switch at t = 0 as shown in the figure. Initially capacitor is uncharged and switch § is closed at t = 0 sec. The graph between Vp — V, s. for variable Resistor R for its three different values s R, , Re, and R; versus time is shown in the fi Ideal Tl, Choose the correct statement ot ene (A) Ri R2> Rs © bi see ‘The two identical rectangular steel frames with the dimensions shown are fabricated from a bar, same material and are hinged, Rectangular at the midpoints A and B of their sides(3m x 1m), frame is resting in the position shown on a horizontal surface with negligible friction, determi velocity v with which each of the upper ends of the frame hits the horizontal surface if the cor¢ Cut. (Take the value of dimensions shown in figure c = 1 m, b = 3/2 m and 0 = 74° i.e., sing =) OF yA ere: cos == & = 10 ms’) £. side view (A) Sms B)8 er (© 3 m/s ©) 4 ms ‘Twelve infinite long wire of uniform linear charge density (2) passing along the twelve edges of a cube, Find electic five nas any face of cube. ae (A) ( at) @) (# Me ox) o(% A sphere is moving towards +ve x-axis with a vel clockwise with angular speed o as shown in the fj instantaneous axis of rotation will be (A) on point P (C) inside the sphere WINIGE Lid, PUTIEE House, 29-A, Kalu Sarai, Sarvapriya Vian website: 37, 38, MPS 6-PHas SIGN Consider a body, shown in figure, consistn : 193 balls, each of mass M connected by a tight 1 t¥° identical ino) MY inpatient tt MO, would be its angular velocity? ends, what (a) VL i (© VBL ® 2a Jey @) VL, If there is a moth sitting at point C of i a rolling sphere, wher ‘ " re velocity of the centre of mass = v, calculate the veloci sitting at point D at this instant "Y of the moth as seen by an observer © (A) V2 vat 45° to the horizontal, 5 5 (© vat 45° to horizontal @) 2v horizontal (©) 2v going down ‘Three simple harmonic motions in the same di : rection having same amplitude and the superposed. If each differs in phase from the next by 2/2 then which ofthe following Gears me (i) Resultant amplitude is (Y2 +i)a a (ii) Phase of resultant motion relative to first is 90° (iii) The energy associated with the resulting motion is 3 times the energy associated with any single motion (A) (i) & (iii) : (8) (ii) only Ce ©) dé Giiy A uniform semi-circular disc of mass m and radius r is suspended as shown in the figure. If T is the time period of small oscillations about an axis passing through point of suspension and perpendicular to plane of disk. Then T is equal to ey Bre A) an [= An ee Bm pa = Ome am Four simple harmonic vibrations yi =8sinot, ya = 6 sin (ot + 7/2), y3=4sin(ot +7), ya= 2 sin (ot + 32/2) dare superimposed on each other. The resulting amplitude and phase are respectively. (A) 4¥2 and tan™'(1) (B) 42 and tar’ (6/8) (C) 20 and tan“'(1/2) (D) 20 and tan”'(2) ‘Two identical particles of charge q each are connected by a massless = is q i are ‘over a smooth horizontal i spring of force consttat A See piece a ee aa They are released en t eek Sat eh iT eanximum extension of the spring ist, the value of k is (neglect gravitational effect) ¢ w Ss Ome © « (D) none of these r ngular accelerati 1 rad/s? about a fixed vertical axis a mass 10gm is placed on the disc at A disc starts rotating with a constant BE ot Cae plane and posing et of son ete he ise and ein 602 TE of 10cm h value of the frictional force on the coin at = Is will be te Iswit'B) 0.01UN (A) 0.02N {b) None of these (© 0.001N GMP-1516-PH-ASSIGN-194 39, 40. 41, 42. 43. 44, 45. WaTIee Lid, FUTJEE House, 2A, Kalu Sarai, Sarvapriya Vihar, New Dele A body is fired with a velocity of magnitude gk Gira) ‘Two particles of mass M and 2 M are at a distance D apart. Under their mutual force they start moving towards each other. The acceleration of their centre of mass when they are D/2 apart is: (A) 2 GM/D* (B) 4 GM/D? (©) 8GM/D* (D) Zero ‘A cork suspended from the bottom of a container filled with water with a string as shown in figure. If the container accelerate in a horizontal direction towards right. Which one is correct? (A) Inclination of string with vertical is tan” (a/g) towards left B) Inclination of the string with vertical is tan” (a/g) towards right (©) Inclination of the string with vertical is x/2 — tan" (a/g) towards left, (D) Inclination of the string with vertical is m/2 ~ tan" (a/g) towards right. Calculate the velocity with which the liquid gushes out of the 4.om® outlet, ifthe liquid flowing in the tube is water and liquid in U tube has a specific gravity 12. Velocity of liquid at point A is veieozymis Arson /20.2 m/s (A) 2.5 m/s (B) 5.5 nv/s (C) 8 ms @) 10 website: wund flljee com. 46. 4 48. 49. 51. GMP-1516-PH-ASsiGN-195 ‘A cone of radius R and height H, is fneans of a string as shown in the fee inside a liquid of on the slant surface of the coneig "force due to the idee (A) pxgHR? ® Z ) apAR? (© =npgHR? 8 3 @) 3 7PEHR Ona cold winter day, the temperature of atm dram shown is made of insulating material agree TC: THe opindrical d of and it contains is latent heat of fusion of ice, p is de rae ab cmc oi, tinea rile etre = r E PLL : 7 Can oe pLH? # © kT Opes pLKT A black body is at a temperature of 2880 K. The ener is obj : gy of radiation emitted by this object with wavelength between 499 nm and 500 nm is wy, between 999 nm and 1000 nm is uz and between 1499 nm and 1500 nm is us. The Wien constant is b = 2.88 x 10° nm-k. Which of the following is correct? (A) m=0 ®) w=0 © w> uw @) > uy ‘A cyclic process is shown in the figure. For the process AB which of the following statement is correct? (A) Heat given to the system will be always -ve throughout the process. (B) Heat given to the system will be always + ve throughout the process. (C) First the heat given is +ve then -ve. (D) First the heat given is -ve then +ve. ‘Two identical rods of length (L), area of cross-section (A) and thermal conductivity kare joined end fo end, If temperature difference of free ends is AT, the heat Qo flows along rods pet second, Find the cr heat flowing per second along the rods if the two rods are placed parallel ‘and temperature tlifference of free ends is AT. There is no heat loss from curved surface, 2 (B) Q=2Q (A) Q=4Q0 i (C)Q= Qos D) Q= QW/2 ‘A cyclic process ABCD is shown in the V-T v diagram. Which of the following curves a represent the same process 2 a - T « ®) GMP-1516-PH-ASSIGN-196, 52. 53. 54. Bar 56. 57. 58. One mole of an ideal monoatomic gas is taken through the thermodynamic process shown in the P-V diagram. The heat supplied to, the system is oy Rv, ( 22) e ev(2) 4 2 x © RY,(1+n) ©) nyi(2) One mole of a diatomic gas undergoes a process P= ——-—;, where F, Vy are constants, The »(¥) translational kinetic energy of the gas when V = V, is given by (A) 2B (B) PoVo 3P, SPN, c) 3PM aA © - Dee A sphere of 8 cm radius behaves like a black body. It is in thermal equilibrium with the surrounding and absorbs 10 W of power radiated to it from surrounding. The temperature of the sphere (= 5.67x10" W/m'K*) is approximately (A) 160K ®) 217K (C) 283K () 347K In the process shown below, which of the following option is correct? (A) AQ py =H, AQye =Hve, AQ. = +ve (B) AQ =+VE, AQge =—ve, AQc, = +VE (C) AQu = He, Cy =, = Hye for AB D) AQy =HVE, AQ: =-ve, AQ, =-ve 2 moles of an ideal monatomic gas is expanded according to relation P# 4 PT = constant from its initial state (P,V,) to the final state, due to which its pressure becomes half of the initial pressure. The change in internal energy is Bi % (A) 3RY, ® 3Ry, os 5 (© 3h, © =n, Ifa monoatomic gas undergoes a thermodynamic process for which its molar heat capacity is equal !0 the universal gas constant. What is the process in terms of V and T? (A) VI"? = constant (B) VT"? = constant (©) V'T’= constant () VT = constant The system shown in the adjacent figure is thermally insulated. An ideal gas is filled in the cylinder having pressure P > atmospheric pressure Po (2 i ‘The spring of force constant k is ly unstretched. The pis S is frictionless. In equilibrium, the piston rises by distance following is not correct. i kx, | mg (A) Final presi ofthe ais (+ Ba. 8) 61 02 SMP-ISIS-PH-ASSIGN-197 (B) Work done by the gas is greater ey than ( Fie + ne.) (C) Decrease in internal ie (1 oO every is ( 's-+mex, +) (D) none of the above A resistance coil, connected to a1 cylinder fted with fictonless piston of mags eqs BMS inside an adiabatic current i flows through the coil, which has resistance’ Rs ideal gas. A piston must move in order to keep the Remco tae The speed with which the (A) Sums Ono will be © mg (D) Data is insufficient. ‘The equation of a state of a gas is given by p(V - b) = expanded from volume V to 2V, the work done 4 ART. If 1 mole of a gas is isothermally luring the process is 2V-Dd| es eri v= (®)RTIn ce V-b (C) RT In pra Vv aV—0) @)RTh| In a sonometer wire the fundamental frequency of the wire is 260Hz. In sonometer the tension is maintained by suspending a 50.7 kg mass. When the suspended mass is completely submerged in ‘water the fundamental frequency becomes 240 Hz. The suspended mass has a volume (A) 0.002 m* (B) 0.0075 m? (C) 0,004 m’ (D) 0.006 m?* A string passing over a smooth pulley carries a stone at one end, while its other end is attached to a Vibrating tuning fork and the string vibrates forming 8 loops. When the stone is immersed in water 10 loops are formed. The specific gravity of the stone is, (A) 2.78 @)18 (C38 20 je resonates in its fundamental mode at a frequency of 200 Hz in Os at a certain ecorucn tte vsontaing 2 moles of O2 and, 3 moles of Ozone is now added to it, then what will be the fundamental frequeny of same pipe at sme tempers? oO ae (D) None of these. The 4" o ee oe vito of the length of the open pipe tothe semi-open ibe ee (D) none of these © 10:3 ae ing medium. Two points P and Q «all directions in a non-absorbing medium. Two poit A point source fs sound ce pee the source. The ratio of the amplitudes of the are at a distance meter waves at P and Q is (B) 925 (A) 25/9 @) 9/5 © 59 : i forks, one of which approaches and the onary observer receives. sound waves from two tuning, beats with frequency 2 Hz. Asai) rslocity. As tis takes place De Scare a Hs and the velocity of GMP-1516-PH-ASSIGN-198 67. 68. 69, 70. 1. nR. 2B. An isotropic point sou fronts, If the sound source is located at the point (1,2,3) , the intensity at point| 4,6, (0+53)], itan ce of sound waves is emitting sound waves of power 314W in spherical waye . mee are measured in meters is:- seot ee (©) 0.25 wim? (D) none of these | itting frequency of sound A man holding a sound source at the top of a tower emitting, 1 © V=1000Hz, Suddenly it slips from his hand and falls down with ‘g’ acceleration. The frequency of sound heard by the man at t= 4 sec. (Velocity of sound=320 mvs, g=10 m/ (A) 860 Hz (B) 888Hz | (©) 894.75Hz (D) 930Hz ‘A motorcyclist going around a circular track of radius 50 m with a speed of 25 mis, is at a point X. A static siren at Y is emitting sound of frequency n. How many times (approximately) in an hour will the motor cyclist hear the —yv y sound of actual frequency Y? (A) 24 (B) 287 (© 600 0) 573 < Two identical loudspeakers are placed on a line at distance 4 m and 6m from a microphone lying on the line joining them. The output of each speaker at the microphone is 2 mW. If the two speakers are connected in series and excited by the same oscillator and they vibrate in phase with each other, the resultant output at the microphone will be (frequency of oscillator = 660 Hz. and speed of sound = 330 m/s) (A) zero @®)2mW (©4mw ©) 8 mw A cylinder full of water is placed just below a container. Water drops come out of the bottom of container at a regular interval of time and strike water surface in tank with velocity vo. If fy is the frequency of sound produced, then frequency of sound received by the observer at rest, is (A) fo (D) data insufficient. Three metallic isolated plates are given charges as shown in figure, the sum of the -3Q, , 4a charges on the outer surfaces of the two plates (1) and (3) is (where -3Q and 4Q are total charges of plates (1) and (3) respectively.) (A) zero ®)Q2 (©-Q2 ©)-2Q Ts) ‘A point charge Q is located at the centre of a hollow spherical conductor having inner radius as Ry and outer radius Ro. The conductor being uncharged initially. The potential at the ae a ae ea (A) KQ| ——+—— sleaal ( [z ral © x9) z| : 1 1 © xo -2] ©) none ofthese. If an electron enters a space between the plates of a parallel plate capacitor at an angle 0, with the plates and leaves at an angle 0, t Fae oi ata E Peer elS thar vue Eevee the plates, the ratio of its kinetic energy while entering © cos*@, ee we 284% ES cos". sin? 0, z One © 2°, WMUUGE Lid, FUTIBE House, 29-8, Kahs Sara z Saneriya Vohar, New Debii-110016 website: wun fjee cc 5 Ph 46 15. 16. 71. B. OMP-ISIE-PH-ASsICN-ing A parallel plate capacitor is char I rged by certain amou the plates is F,. If regi kel i eh TA Meter (w te Correct et tm a tawen sae 0e of interaction © ®) Fer, (D) none of these AA solid hemispherical uniformly charged body having charge Q is kept symmetries along the y-axis as shown in the figure. ‘The net electric field at (4, 0, 0) is : (0, 0, 0) (4,0, 0) ®) less then x? a0 1 2Q : O>— 2 mice) iz 4ne, d’ 4ne, & ©) meet ‘The amount of work done in assembling a system of maximum number of charges each of magnitude so that each charge is equidistant from the other is equal to 7 : q 3q° ae aaa © 2 of net (D) none of these. A tiny electric dipole of dipole moment p is placed at a distance r from an infinitely long wire with itsp normal to the wire. If the linear charge density of the wire is , the electrostatic force acting on the dipole is equal to AP ap ® — ® 4ne 0 Megr 20.P Oe, ON mae A solid here of radius R has a spherical cavity of radius r as shown in the figure. If fete charge density o is uniformly distributed over the whole volume of the sphere, then electric field strength at the centre of the sphere will be > Ro Ro oes (A) ———— : aes (R-1) 3ne, ay eine Ons OR Ry 36,(R-1) “The potential on the Nib shell due to N concentric shells having charges Q, 20, 3Q, NQ and radi 2a, 3a,.....Na respectively is ey tae Ree ® QNIN+)) he nea (Gj (D) none of these 2m a Ti 0100000, 265609, ma Serge Vir, New Dedi-1 10016, Lid. PUTUEE Tiouse, 20°, Raha Sara, Sarvapriya Vina, how Po (GMP-1516-PH-ASSIGN-200 81 82 83. 84, 85, 87. A non conducting ring of radius R, is charged such that the linear charge density is 2. cos? 9 Where g 's the polar angle. If the radius is increased to R, keeping the charge constnat,the linear charge densi is changed to, cos* @ . The relation connecting R,,R, %,and2, will be a @) A,/R, =2,/R, (AIR, Ora, = ©) UR, =22R, A Point charge q is placed inside a conducting spherical shell of inner radius 2R and outer radius 3R a distance of R from the centre of the shell. The electric potential at the centre of shell will be i 14 @ —2 ed 4xe, 2R ©) a, IR ees Qe (yet 4x, OR 4ze, 3R In the given circuit, the potential difference across the capacitor is 12V. Each resistance is of 30. The cell is ideal. The emf of the cell is (A) Isv (B) 9V © Rv (@) 24V Find effective resistance between A & B. 19 0 (a) 2a ®1aQ (872 @M)65Q 4 = 1s 10 12 A milli-ammeter of range 10 mA and resistance 9 Q is joined in a SOA oma. circuit as shown in the figure, The meter gives full scale deflection, when current in the main circuit is ‘T’ and A and D are Be used as terminals. The value of I is MoT] Be | 06a | 090 heen, E (A) 1.094 ) 109A clea (©) zero. (©) 0.109 A A potential of 400 V is applied at the point A. The value of resistance R; = 10000, Rz = 2000 and R; = 1000 © are connected between points A and G as shown in the figure. Point G is earthed. The measured potential difference by an ideal $® voltmeter connected across Re is (A) 100V ®) 200 Vv R (© 300V @) 400 V R;3 Ina parallel plate capacitor, the region between the plates is filled by a dielectric slab. The capacitor is charged from a cell and then disconnected from it. The slab is now taken out (A) the potential difference across the capacitor reduces. B) the potential difference across the capacitor increases. (© the energy stored in the capacitor reduces. (D) no work is done by an external agent in taking the slab out. 88. 89, 91 pes 94, SMP-1SI6-PH-ASsIGN 201 In a regular polygon of n Sides, each co magnitude Q are placed at (n— 1) comers Ty, 5% 8 distance + f . The magni rom the centre, ident kK \Bnitude of the fie identical charges of we fyi ees is i @ M=KQ r n-1 ©) =o where, k= 1/4ne, In the circuit shown the cells are id capacitance of the capacitor is C and is as Sri nado C ~ R. The switch X is first connected to Y and then to Z. After a ike time, the total heat produced in the resistor will be es 5 x (A) equal to the energy finally stored in the capacitor. : (B) half the energy finally T stored in the capacitor. ae (©) twice the energy finally stored in the capacitor. a eae () four times the energy finaly stored in the capacitor. In the arrangement shown in the figure when the switch S, is open, the galvanometer shows no deflection for ¢ = L/2, When the switch S, is closed, the galvanometer 0Q shows no deflection for £ =Su. ‘The internal resistance () of 6 V cell, and the emf E of the other battery are respectively (A) 30,8 V (B) 20,12V (© 20, 24V @) 39, 12V a ve a Ye for given figure in steady state is Ee ee ©) zero Vv 3 wh] Pee ¥ ice a A hemi-spherical network of radius a is made by using a conducting wire mo * resistance per unit length r . The equivalent resistance across OF is of resis 5 30042) aS eer Ae gs aD > © ra(n+4) © raeiee . ‘ P, a itch S is closed is ‘The charge flown from P to Q when the swi! (B) 6pC Ly (A)3pC. (p) 15nC (© 9c ‘ =0, The K is switched onat t= ven cirouit diagram, the key B ISS att = oo wil ae ren Girent though the cell at 13 (A) 3:1 (p21 ©@12 : | 95, 96, 97. 98. Ba: The location of three long wires carrying same current I are x = ~a, X= 0 and x = + a respectively as shown in figure. The magnetic field at point (0, 0, a) is (Ay tel tl a ©) Pra I jee ira OE A steady current is flowing in a circular coil of radius R, made up of a thin conducting wire, The magnetic field at the centre of the loop is B,. Now a circular loop of radius R/n is made from the same wire without changing its length, by unfolding and refolding the loop, and the same current is passed ae it. Ifnew magnetic field atthe centre of the coil is Bc, then the ratio By/Be is. Tin ®) ib © nn ‘Or A wire of cross-sectional area A forms the four sides of a square and is free __ to tum about an axis OO’. If the structure is deflected by an angle 0 from © the vertical when current i passes through it, in a magnetic field B acting vertically upward and density of wire is p, the value of 0 is given by foes / 2Apg 2ApB (A) ——== eee (A) SFE = coto ® Bom iano Ape _ Apg _ © =BE= sino ©) SE cos0 ‘The magnetic field at the centre of the circular loop as shown in the figure, where a single wire is bent to form a circular loop and also extends to form straight sections wal wel we me we @) Mtv a jo 1 Holy 1 bo 1 al =o) il oor ( ais) 2 tan) Only circular part of the wire shown in the figure has resistance o per unit length. The radii of the circles are r and R and ZPOQ = o. (in radian). If a potential difference Vp ~ Vg = V is applied across the point P and Q, the magnetic field at point O is HY ne] HV[ a 2n—a wn ee Ola tev al stor lia one R re eal The magnetic field strength at a point P distant r due to an infinite straight wire carrying a current iis as shown in the figure (A) Ho 8) poi/ V2 (©) (oiNl2xe ©) fo+ V3] Ane’ 103 104. 10s. 106. A charged particle of mass m & ¢ field B with a velocity v making having width d, when the moan Particle penetrates h: “@ = (sie [1-23 Laas Bia™ A charged particle moving along +ve x-direction region where there is a uniform magnetic field B, eeoes The particle gets deflected at an angle 0 from its initial path. The speci ks The fi charge ofthe particle is (specific charge isthe charge per unt ass) aes! axed yeos@ 7) vtan® a Bd aera v i mo vsind ta real A particle of charge q is projected from point P with velocity v as shown. Choose the correct statement from the following. : oP iad (A) Magnetic field at P is Fe [a+] ® (B) The magnetic field at P is poi 2nr. (©) The particle initially accelerated towards wire A. (D) The particle initially accelerates towards wire B. ‘Two infinite wires, camying currents i, and is, are lying along x and y y axes, as shown. In the x-y plane, is (A) locus of point where B is zero is a circle. @) locus of points where B is zero isa line —y (O) B decays hyperbolically along any line parallel to x-axis (D) B decays hyperbolically along any line parallel to y-axis In the figure shown a coil of single tum is wound on a sphere of rads and mass m, The plane of the col i parallel tothe inclined plane and lies in the equatorial plane of the sphere. If sphere is in rotational equilibrium the value of B is (current in the coil is 1) : cw mesine A GMP-1516-PH-ASSIGN-204 107, 108, 109, 110. 1. 112. 113. Three particles electron (¢), proton (p) and Helium (He) are moving in circular paths with speeds in the x-y plane in a region where a uniform magnetic field B exists along z-axis. The fae taken by e, p and He inside the field to complete one revolution is t,t, and ty. respectively, then" (A) te (B) te> ty> be © tie (D) none of these A long cylindrical wire of radius ‘a’ carries a current i distributed uniformly over its cross-section, jp the magnetic field at distance r and R from the axis have equal magnitude, then, (a= Ret (®) a=VRr © a=Rr/Rer (D) a=R’/r A square conducting loop of side L is situated in gravity free space. A small conducting circular loop of radius ‘r’ (r< inductance, ir ind it (A) 02, o = angular frequency ®) een = (© R’, R= resistance, (D) none of these. A radioactive sample undergoes decay as per the following graph. At 1B time t = 0 the number of undecayed nuclei is N.. Calculate the number of nuclei left after one hour, icy ®) N/e” Os ©) N,/e" SATESS TOS tein) Binding energy per nucleon for C'? is 7.68 MeV and for C’? is 7.74 MeV. The ired remove a neutron from C'? is eee (A) 5.49 MeV (B) 8.46 MeV (© 9.45 MeV ©) 5.49 MeV ‘A radio nuclide A; with decay constant 2; transform into a radio nuclide Az with decay constant ‘2. ‘Assuming that at the initial moment the preparation contained only the radio nuclide A:, then the time interval after which the activity of the radio nuclide A» reach its maximum value is In(./2a) In(2/2) yy dam 2 nay (©) In@iz- 1) (D) None of these \d the number of nuclei in the sample if initially there A radio isotope *X’ has a half life of 10 seo. Fine ‘est from a height of 3000 m; when it is at a height of are 1000 x 10 isotopes which are falling from r 1000 m from the reference plane (A) 50 x 10” (B) 250 10% (©) 29 x 10” (D) 100 x 10° 1f 10% ofa radioactive material decays in 5 days, then the amount of the original material left after 20 days is approximately: (A) 60% @) es (©) 70% () 75% A free nucleus of mass 24 amu emits a gamma photon (when initially at met), The exeay of the photon is 7 MeV. The recoil energy of the nucleus in a in (Assuming 1 amu = leV). (A) 22 i ©31 D) 2 The de-Broglie wavelength of a thermal neutron at 927°C is 2. Its wavelength at 327°C will be (A) 02 @ 2 (© 242 (D) 20 — ee TIE ane Deana caren Suoaprye var, New Da 110019 : mele: une ies om 141, 142 143, 144, 145. 146. 147. 148. 149. GMP-1516-PH-ASSIGN-208 The electron in a H— atom makes a transition from m —> ms Where my and ny are the principal ‘number of the states, Assume Bohr's model to be valid, The time period of the electron in initat is 27 times that in final state Find the possible value of m; and ny if only just first 10 states are jr visible range of instrument. z (A) 1-2, 2-4, 3-6, 4-8 5-10 Se (© 14,28 eae If E;, Ez and E, represent respectively the kinetic energies of an electron, an alpha particle ang , Proton, each having same de-Broglie wavelength, then: (AVE) > Es> E> (B) E,> By > By (C)E,\>E)> Ey (D) E, = E= Ey The ionisation energy of the ionised sodium atom Na’"” is (A) 13.6 eV (B) 13.6 11 Ev (© 3.6/1) ev (D) 13.6 x (1?) eV A point object is placed in front of a silvered plano convex lens of refractive index n radius of curvature R, so that its image is formed On itself. Then the object distance is equal to (A) R/2n (B) 2R/n (Rn (D) none of these 2 There are n number of radioactive nuclei in a sample that undergoes beta decay. If from the sample, n' Inumber of f particles are emitted in first 2 seconds, than half life of nuclei is wR (B) 0.693 x 2n/n') (C) 0.693 Inn/n') (py 222.693. The shortest wavelength produced in an X-ray tube operated at 0.5 million volt is (A) dependent on the target clement (B) about 7.5 x 10° m (©) double of the shortest wavelength produced at 1 million volt (D) dependent only on target material If the Ky radiation of Mo (Z = 42) has a wavelength of 0.71 A’ find the wavelength of the corresponding radiation of Cu (Z = 29) AT Ae (B)2 Ae (C) 1.52 A? (D) 1.25 Ae A beam of electrons accelerated by a large potential difference V is made to strike a metal target 10 produce X-rays. For which of the following values of V, will the resulting X-rays have the lowest minimum wavelength? (A) 10kV (B) 20 kv (C) 30kV (D) 40 kV ‘An experiment measures quantities a, b, c and x the value of x is calculated from x = able’. If the maximum percentage error in a, band c are 1%, 1% and 2% respectively, (A) the error in x may be zero (B) for any particular reading, error in x may 10%, (C) maximum percentage error in x is ~4%, (D) maximum percentage error in x is 8%, 150 151 152 153. 154, 155, 156, GMP-1516-PH-ASsIGN-209 A wire of lengih ¢ = 6 + 0°06 cen and wine ¢ Maximum percentage error in density is 88 # 0.005 em and mass m= 03 0.003. gum. as @2 © ©) 68 {A student performs an experiment for determina fmt ‘rmination of g| ace pon Im, and he commits an error of Af, For T he takes the time of oscillations withthe stop wate! ch of least count AT human error of .1se. For which ofthe following data the me “igeue eens -asurement of g will be most accurate? At aT n Amplitude of oscillation (A) 5mm 0.2 sec 10 5mm (8) Smm 0.2 sec 20 5mm (© 5mm 0.1 sec 20 1mm (D) 1mm 0.1 sec 50 1mm Which of the following does not have the same dimension? (A) Electric flux, Electric field, Electric dipole moment (B) Pressure, stress, Young’s modulus (© Electromotive force, Potential difference, Electric voltage, (D) Heat, Potential energy, Work done ‘Two uniform solid cylinders A and B each of mass 1 kg are connected by a light spring of force constant 200 N/m at their axles and are placed ‘on a fixed wedge as shown in the figure. The coefficient of friction between the wedge and the cylinders is 0.2. The angle made by the line AB with the horizontal, in equilibrium, is (A) B® 1s © 38° (D) None of these. A person wants to drive on the vertical surface of a large cylindrical wooden ‘well’ commonly known as dealrell ina circus. The radius of the ‘well’ is 2 meter, and the coefficient of friction between the tyres of the motorcycle and the wall of the well is 0.2. Minimum speed the motorcyclist must have in order to prevent slipping should be (take g = 10 mvs") a SGeT @®) 15 m/s © 20 mis Cee a re i 2 By applying a force F =(3xy~5z)j+4zk a particle is moved along the Fan =¢ = x? from point (0, 0, 0) to the point (2, 4, 0). The work done by the F on y the particle is 140 (0,0,0) x » = sy 192 © re Velocity oa body of mass 2kg moving inx- plane is given by V = (2i+- fl) mis, whee tis theme in second, The power delivered to the ‘body by the Ta ‘on it att = 5 sec is. (A) 80 m/s? Ee © 400s? 157. A solid sphere of mass m is lying at rest on a rough horizontal surface: ‘The coefficient of friction between ground and sphere is}. The maximum value of F, so that sphere will not slip, is equal 10 (A) Tumg/s (B) 4umg/7 : (© Sumg7 (©) 7amg/2 158. A block of mass V3/10kg is placed on a rough horizontal surface as shown in the figure. A force of 1 N is applied at one end of the block and the block remains stationary. The normal force exerted by the surface on the block acts (g = 10 m/s") (A) through the centre of mass of the block. (B) through point A. (©) through point B. (D) through the point at a distance 5 cm. from A. 159. Consider an infinite matrix of capacitors as shown in the figure. ae effective capacitance between point A and B will be S (B) 2c © (D) can’t be determined. 160. 161. 162. 163. GMP-1516-PH-ASSIGN-210 Consider the circuit shown in the figure. Taking the potential to be zero at the negative terminal of the battery: (A) Potential at F will be higher than the potential at C (B) Current in AF will be greater than that in BC (©) Direction of current in CF will be from C to F (D) Current in FE will be greater than current in CD ‘A certain amount of a diatomic gas undergoes a certain polytropic process in which it absorbs 100 Joule heat and performs 25 joule work. The process can be described by the formula: (A) PV5=C ®) PV'S=C (© Pv'*=c () PV*5=C ‘The figure shown the cross section of a long cylindrical conductor through which an axial hole of radius r is drilled with its centre at point A. O is the centre of the conductor. If an identical hole were to be drilled centred at point B while maintaining the same current density the magnitude of magnetic field at O (A) will increase (B) will decrease (C) will remains the same (D) May increase of decrease depending on the value of r 'A circular conducting loop of radius R carries a current 1. Another : Straight infinite conductor carrying curent I passes through the diameter 1 ef this loop as shown in the figure, The magnitude of force exerted by 5 the straight conductor on the loop is W (A) pol” ®) pol? Bol? PE 2 ie o 164 165. 166. 167, 168, 169. positive charge is moved froma point ( qoN™ i the figure. A point le +s 7 toa point 2 it ice from the negative plate of C. aE cz at lee it J lates of Cy. Then the work done by there ta the distance between (A) positive me by the electric Gis eal (C) zero, live & (D) data insufficient, ‘An RC circuit is made using five identical c which can withstand a maximum voltage FED maximum voltage which can be safely applied across points A and Bis =f © uy (B) 3V ee % ) ( @)1V Weal Eo ‘A sonometer wire has a length133 cm between two fixed ends. Two bridges are placed between the fixed ends so that wire divide in three segments whose fundamental frequencies are in the ratio 1: 3:4 ‘The distance between bridge it. (A) 28cm (B) 24cm (©) 420m ©) 36cm ic A point mass m is displaced slightly from point O and released. It is constrained to move along parabolic path having equation x? =ky then its angular frequency of oscillation is: 28 pe ® i OK k 3B, ig @) J ©, J% Ody 2k sherical shell is cut into two pieces along a chord and ae by large distance as shown in figure. For points A and B, where E and V are gravitational potential and ‘ field respectively (A)V, =p and |E, PLES | @) ¥,<¥% and |E, HIEs | (COV, =Ve and JE, Es | ) VY, > Vp and |E, IE | When is i mes, the short wave limit of the ied to an X-ray ‘tube is increased 15 times, aes seiner shifts by 26 pm. Find the a salaee applied to the tube. (A) 16KV CO (© 2KV ‘GMPOI516-PH-ASSIGN-212 170. 171 172, 173. 174. 175. Find the minimum radius for a planet of mean density p and temperature T which can detain o its atmosphere, (My = molecular weight of Oxygen and G = Universal Gravitational constant) [2 t 9 RT iM,px ©®) 3 Gaon 1S RT is RT © J— ocean V9 GM,px ©) is cmp A cubical frame is made of 12 rods each of mass m and length ¢. The moment of inertia of the cube about its face diagonal is xyBEn in, oles EA Sos (A) me (8) aoe 43 23 © Bime Bae © é mé' () i me ‘Two thin rods are moving perpendicularly as shown in the figure. If the y friction acting between them is Fr then the unit vector in the direction of friction force acting on the rod lying along x-axis is -3) (3) Vs @) G (3i+2i) Daas (D) none of these Two vertical plates submerged partially in a wetting liquid form a wedge with a very small angle 83. ‘The edge of this wedge is vertical. ‘The density of the liquid is p, its surface tension is T, the contact angle is 0°, The height h, to which the liquid rises, as a function of the distance x from the edge is 25 20 © ex) ars 27 (60) fe Oe © Se Rank the magnitude of B.dl for the closed paths shown in figure from the smallest to largest. (A) a,b, d (B) a,c,d,b ©adeb @)acbd Two coherent sources S, andS, are kept on the edges of a step as shown in the figure. An infinitely long screen is placed on the right side of sources and lies along y o’7 plane. Calculate total no. of maximas observed on. the soreen. PS, =84 andPS, = 6A . Where Ais wavelength of light used. (There is no reflection from steps) (A) 18 ®) 16 Ou @)12 176. 177. 178. 179, 180, a a GMP-1516-PH-ASsIGN-213 There are two identical square metallic plates kept on a roug) et horizontal floor at t= 0, plates are given ° given that plate 1 is rotating about its ee © and itis oy about one of its corner O, If t, and t, are the erie ls ae n by both : plates to come to rest then 4 is (Both tee 3 are independent cases) aye a ®)2 sak og 4 @ A Intensity due to a Tine source §) at a distance of ris Ip and Line souree intensity due to other line source $3 at a distance of ris 4lo and frequency of both source are equal. If interference occurs at a point P as shown in the figure and wavelength of both sources is r, find the resultant intensity (in swatl/me) at P. (where Ip = 2 watvm?) (A 8 (2 Line source $2° ee @)1 Ar {A ring of radius R having total charge Q uniformly distributed over the ring has a charge particle a kept at its centre, Ifa transverse pulse is created on the ring, then find the time when pulse completes ‘one revolution, (Total mass of ring is m) ‘Two dipoles P, and P, are oriented as shown in the figure. Assuming dipole of dipole moment P, to be placed at origin and dipole of moment 8 isata distance ‘d” from origin then ed Gene ePrta (A) Torque experienced by Fis ~ gage * ee mlaeP Rs (®) Torque experienced by B iste ge ; fae enh (©) Torque experienced by P is reg 1 RBG (D) Torque experienced by P, is+ ree cated in a coil of resistance R due to a charge q passing through it if What amount of heat will be gener 4 pro uniforly thing a time interval AC? the current in the coil decreases down ? ore a te 3 At 3 At an a Oya GMP-1516-PH-ASSIGN-214 181, 182, 183, 184. 185. 186. 187. A certain amount of ice is supplied heat at a constant rate for 7 minutes. For the first one minute thy temperature rise is uniformly with time. Then it remains constant for the next 4 minutes and again the temperature rises at uniform rate for the last two minutes (Given Specific heat of ice = 0.5 cal/g.og, atent heat of fusion = 80 cal/gm and specific heat of water = 1 cal/gm-°C) (A) The initial temperature of ice is 30°C. (B) The final temperature at the end of 7 min is 40°C. (©) The final temperature at the end of 7 min is 30 °C (®) Between t= 1 min to t= 5 min it does not changes its phase. The displacement of two identical particles executing SHM are represented by equations x; = 4 sin [10 * (@/6)) and x2 = 5 09s ot. For what value of o energy of both the particles is same? (A) 16 units (B) 6 units (© 4 units (D) 8 units A gas for which y = 4/3, is heated at constant pressure. What percentage of the total heat supplied is used up for external work? (A) 12.5% (B) 25% (©) 35% D) 70% ‘Two sound sources shown in the figure vibrate in phase. By moving S; along PS, consecutive minima are heard when L; ~ Lz has values, 20 cm, 60 cm and 100 cm. What is the frequency of sound source if Be the speed of sound is 340 m/s? eg (A) 280 Hz (B) 330 Hz & (©) 850 Hz (D) 440 Hz A block of mass m is attached to a spring of force constant k whose other end is fixed to a horizontal surface. Initially the spring is in its natural length and the block is released from rest. The average force acting on the surface by the spring till the instant when the block has zero acceleration for the first time is k mg 2mg Ces @) a 4 (© 2me @) —= % % The gas inside a spherical bubble expands uniformly and slowly so that its radius increases from R to 2R. Let the atmospheric pressure be p and surface tension be $. The work done by the gas in the process is 2 3 a 2a Pare ie 2m Vee 25mp,R? , 23nSR? (Cys (D) None of these 3) 2 A thick rope of length 2L and linear mass density 9, is joined at B to a thin rope of length L and linear mass density 41, The system is horizontally stretched by the two vertical wall A and C. Assuming B to be a node, find the minimum number of loops in the thick rope. wl Oe ae 4 188 189. 190. 191 192 193, GMP-1516-PH-ASSIGN-ats. In the figure shown surfaces are sm have same mass m, The two icles blocks released. It is found that bodies 'splaced right and equation xa A sin (0t#4), xp =2A sin (r+ gy ie A block of mass m lie on a horizont tal smooth. ration and connected with the springs at their cra eat eee as shown in figure, When block slightly displaced then find the time period of oscillation. eit Ox ®) » [2 7k © af p) = [= 7k CONTE If the radius and surface tension of a spherical soay i uniformly distributed ever the outer surface of te bbl is eae to dele is ads (Gon et atmospheric pressure is Po and inside temperature of the bubble during expansion remains constant.) (A) 8nr[eor (7Por + 12T)]'? (B) 4nrfeor (7Por + 12T)]'? (C) mtfeor (7Por + 127)? (D) 8nrfeor (7Por + 12T)}"° Charge Q is uniformly distributed on the rim of a thin insulating disc of mass q ,»+** m which is initially at rest and placed on a smooth horizontal surface. What 7 will be the angular velocity of the disc if a magnetic field B, perpendicular to the plane of the disc is switched on? S (A) QB/2m (B) QB/3m ae (© 2QB/3m @) QB/m ite kinetic energy of an electron so that its de-Broglic wavelength is What should be the approximat ; . 3 equal to the wavelength of X-ray of maximum energy, produced in an X-ray tube operating at 24800 V? (given that h= 6.6 x 107 joule-seo, mass of electron = 9.1 x 107! kg) (A) 600 ev B) 36.50V (D) —300eV (C) 6000 eV A broad metal plate is connected to earth through @ conducting wire as shown. An electron flies with constant velocity along a straight line above the plate at a distance much less than the linear dimensions of the plate. I the current I flowing from the earth to the plate is pose to be positive, then which of the following graph correctly depicts the variation of current I with time? 7 TASC oe (B) Se ee : (ae i SI Dat 1001 gaa, 2OSIAAY, PS “THUeE one Dea Ta So Sea ee a GMP-1516-PH-ASSIGN-216 194, 195. 196, 197. 198. © 1 o ! A long solenoid of radius 2R contains another coaxial ©°Re°?eeoeeeeeeeovay solenoid of radius R. The coils have the same number of turns per unit length and initially both carry zero current. At time, t= 0, current start increasing linearly with time in both solenoids. At any moment the current flowing in the inner coil is twice as large as that in the outer one and their directions are same. A charged particle, initially at rest between the two solenoids, start moving along a circular trajectory due to increasing current in the solenoid as shown in the figure What is the Tadius of the circle? (Assume magnetic field due to each solenoid remains uniform over its cross- section.) : (A) VaR @) VR © 3R (D) none of these oovgocehdoovccveey eoeee A particle moves along x-axis. The position of the particle at time t is given as x = f ~ 917 + 241+ 1 Find the distance traveled in first 5 seconds (A) 20m B) 10m (©) 18m (D) 28m A particle is moving in an isolated x-y plane. At an instant, the particle has velocity (4i+4}) m/s and acceleration (3i-+5j) m/s, At that instant what will be the radius of curvature of its path? (A) 162 m @®) ism (C) 18m (D) None of these ‘There are two incline planes AO and OB inclined at 45° and 60° respectively with the horizontal as shown in the figure. When a man moves on the inclined plane AO, he is observes that the rain drops are falling 45° with the vertical. When the man moves on the inclined plane OB, he observes that the raindrops are falling vertically downward with the horizontal. Then actual speed of rain is (A) 5V2.mis ®) 203 ms (©) 102 ms (©) None of these In the given situation disc and ring are connected with a string as shown in the figure, Both are placed on the rough surface of coefficient of friction 1. A force F is applied on the centre of disc horizontally. Then frictional force acting on the ring will be a) 2E ®t AS De (©) Zero ©) None of these, GMP-1516-PH-ASSIGN-216 194, 195, 196, 197. 198. © 1 o ! A long solenoid of radius 2R contains another coaxial ° solenoid of radius R. The coils have the same number of tums per unit length and initially both carry zero current. At time, t= 0, current start increasing linearly with time in both solenoids. At any moment the current flowing in the inner coil is twice as large as that in the outer one © © and their directions are same, A charged particle, initially at rest between the two solenoids, start moving, along a circular trajectory due to increasing current in the solenoid as shown in the figure What is the radius of the circle? (Assume magnetic field due to each solenoid remains uniform over its cross- section.) 3 (A) VaR @®) VBR © SR (D) none of these A particle moves along x-axis. The position of the particle at time t is given as x = t* — 9° + 24t + 1, Find the distance traveled in first 5 seconds (A) 20m (B) 10m (©) 18m (D) 28m A particle is moving in an isolated x-y plane, At an instant, the particle has velocity (41+4}) m/s and acceleration (31+5}) nvs*, At that instant what will be the radius of curvature of its path? (A) 16V2m ®) 15m (C) 18m @) None of these There are two incline planes AO and OB inclined at 45° and 60° respectively with the horizontal as shown in the figure. When a man moves on the inclined plane AO, he is observes that the rain drops are falling 45° with the vertical. When the man moves on the inclined plane OB, he observes that the raindrops are falling vertically downward with the horizontal. Then actual speed of rain is (A) 5V2 mis 8) 203 ms © 102 ms (D) None of these In the given situation disc and ring are connected with a string as shown in the figure, Both are placed on the rough surface of coefficient of friction . A force F is applied on the centre of disc horizontally. Then frictional force acting on the ring will be 2F (A) 3 (C) Zero OMPasi6-rH-assicn217 199, A ball is thrown from point A ma . taking an angle 60° with line OAS as shown in the figure, where OA = 2. (0 is the ke eel z Centre of spherical surface of POS} after striking the hemi-spherical parallel to OA. The coefficient of resaietes Ca eomunas in direction surface is [Neglee the effect of gravity any type of icions ea @) ¢=0.75 (D) None of these 200. A ship oe ‘mass m is anchored in the middle of a river and water is jowing with a constant velocity vo. The horizontal force exerted on the ship by the anchor chain is Tp. ne cd ca suddenly breaks, determine the time required for the ship to attain a velocity equal to 0.5 yo. Assume that the frictional resistance of the water is proportional to the velocity of the ship relative to the water. ay (A) 2in2 mv, In: % ® 5h? 3mv, Oa m (D) None of these 201. _ A triangular rigid wire frame ‘AOB’ is made, in which length of each wire is € and mass is m. The whole system is suspended from point O and free to perform SHM about x-axis or about z-axis. When it x performs SHM about x-axis its time period of oscillation is T; and when it performs SHM about z-axis, its time-period of oscillation is T;, then choose the correct option. (ATi T2 im e On=T (D) None 202, A rod of mass m and length L is kept on a horizontal smooth surface, The rod is hinged at its one end A. There are two springs, perpendicular to the rod, kept in the same horizontal plane. The spring of spring constant k is rigidly attached to the lower end of rod and spring of spring constant 12k just touches the rod at its midpoint C, as shown in the figure. If the rod is slightly displaced leftward and released then, the time period for small oscillation of the rod will be [m 3x [m oe 3k O ax 3n{ fm, So 3 = /— —— | | Oy | ax 4 =| 203, ‘The figure shows an equiconvex lens (of refractive index 1.50) in contact with a liquid layer on top of a plane bee sinall needle PQ with its tip on the principal axis is mov along the axis until its inverted image is found at the position of ihe needle, The distance of the needle from the Tens is vated io be 43.0em. The Hauid 1 raoral aie He experiment is repeated. ‘The new distance 1s measured to be 30.0cm. The refractive index of the liquid is ae (A) 5/4 a 8 © 43 meee Var, New Dein 110016, Fh 46105000 Basen, ste: wi fjee.com TMUUGG Lid, FITIEE House, 29-A, Rahs Sara, 92 GMP-1516-PH-ASSIGN-218 204, 205. 206. 207. A brass boiler has a base area of 0.15 m? and thickness 1.0 cm. It boils water at the rate of 6.0 kp/min when placed on a gas stove. The temperature (approximately) of the part of the flame in contact with the boiler is (Thermal conductivity of brass = 107.4 Js'm''2C"', Heat of vaporization of water = 2956 x 10° Skgty, (A) 2400 K (B) 240 °C (© 2400 °c (D) 24000 °C A non conducting rod AB of length/3R , uniformly distributed charge of Q linear charge density 2 and a non-conducting ring of uniformly distributed charge Q, are placed as shown in the figure. Point A is the centre of ring and h Tine AB is the axis of the ring, perpendicular to plane of ring. The VBR 8 electrostatic interaction energy between ring and rod is, OH Qe ea [By Bus f ( Naren 3) () ase Ca 3) Qn Qn © Frag MWD 7, ene - 3) The entire network shown here, has uniform wire having resistance per unit length as 1 Q/m. It is placed in a uniform but time varying magnetic field 8 (directed into the plane). If the strength of the magnetic field is increasing at 1T/s, the ratio Iy/l2 equals x x im x x (A)2 (4 © ©) 146 ‘A block of mass m = 5 kg is placed on the wedge of mass M= 32 kg as shown in the figure, Find the acceleration of wedge with respect to ground, (Neglect any type of friction. String and pulley are ideal) ‘i 3 —m/s* = ms? apm ®) | ms 4 3 =m = ©qzm ©) & ws GMP-1516-PH-ASSIGN-219 gos. A small collar of mass m is given an initial horizontal circular track fabricated from a sien go, mite ; ni from a_slen ete kinetic friction is jx, determine the distance uavelled vote goose of to est. (Recognize thatthe friction force dene fore the collar comes nds on the net normal force), 2s aeons Wy pl veer? a) of Sere Btn) EG HG L iy 9 Vp +/V6 +170? (© 2a] oa r | ve + +o (D) =—h Me m9 PETA cia 209. An ideal gas undergoes a circular cycle as shown in the figure. Find the e maximum temperature of cycle to minimum temperature ofeycle. & wy (142) 24 BY mes meg 342) 7 of) o(i3) 210, A particle is projected from point P on inclined plane OA perpendicular,to it with certain velocity v. It hits another inclined plane OB at point Q perpendicular to it, Point P and Q are at hy and hp height from ground. (>) (A) hy = hy (B) hy > hy (Oh 40N {(B) Cube will not perform any type of translation or rotation motion if 10 N i Obrb>h Four identical conducting rods : are velocity of point R is v. A uniform Wiper iby pins. The the paper. Choose the correct statements, (PQ) oe out of QR=SR=30 See, (A) The motional emf across PQ is 884 (B) The motional emf across PQ is 2B (C) The motional emf across QR is 2B 5 (D) The motional emf across QR is SY 5 A truck has to move to a diametrically opposite point on a circular track Bs which surrounds a field. The speed of the truck along the track is 2vo. While that in the field is vo. ‘The driver plans to move along an arc of a Fi circle and then along a straight line as shown, (A) To reach P in shortest time, Q must be equal to 60°. & R[x —|=+V3 |. [§ : 4] (B) The minimum time required to rach P it Yo (C) The distance travelled to reach P in shortest time is ne (D) The angle 6 will not depend on the value of Vo. {A body of mass m is moving along a circular path of radius R such that ts kinetic energy at any instant Saal wuan(?) where to is an appropriate constant, Then : (A) The magnitude of tangential component of force acting on ust be constant. (B) The magnitude of centripetal force acting ‘on the body will be directly proportional to ty (©) After a long time, the resultant force will make a very small angle with the ae (D) The magnitude of centripetal force acting on the body will be directly proportional to t Feat ‘Att=0, a force F= at? is applied to a small body of mass m at an angle a resting on @ smooth horizontal plane a (A) velocity of the body at the moment it brakes off the plane iS fora? sina.” : fe ca. s MORES (the distance travelled by the body before breaking off the plane 5 $5 Sinc-tana off the plane is g cot 0. (© Its acceleration at the time of breaking B mg (D) Time at which it breaks off the plane 8 JF Sing. GMP-1516-PH-ASSIGN-230 10, M 12, 13 14. Masses m, 2m, 3m, 4m, .....:.0..:.sse14 12 m are kept on the peripheri of a clock having a circular dial. The masses are kept such that m is kept at 1, 2m is kept at 2, 3m at 3.0.0... 12 mat 12, g is the gravitational field intensity at the centre of dial. (A) At 9:30 the hour hand is in the direction of G (B) At3 : 30 the hour hand is in the direction of § (©) there will be no component of G along hour hand at 6 : 30 (D) There will be no component of § along hour hand at 12:30 A charge is placed at rest in a region of time varying magnetic field. After a short time (A) The charge will experience only electric force. (B) The charge will experience a magnetic force (C) The charge wil experience an electric force. (D) None of the above. A block of mass m is tied to a string of length L and the ‘assembly is kept on a smooth horizontal table as shown in the if figure. Now the other end of the string is moved with a rm constant velocity vo j. (xy plane is the plane of the table) Then (A) m will execute an oscillatory motion. (B) m will follow a circular path as seen fro the ground. (C) Angular momentum of m will remain conserved about the other end. (D) Angular momentum of m will remain conserved about the origin. [¢~—— . ——> Angular momentum of a planet about the point of it's apogee is (A) Maximum when the planet is at perigee. (B) Minimum when the planet is at apogee. ; (C) Maximum when the planet is at end parts of the minor axis of the ellipse (D) Minimum when the planet is at the end points of the minor axis. A certain rough platform perform oscillating given by y= A sin at }, while another block kept on the platform perform oscillations on the platform given by = t) 7. As seen from, the ground. a) hee ee ch of fitional force on the block wil always be opposite to its velocity relative tothe rm. ®) aa rerretion of frictional force on the block will be always opposite to its relative acceleration wart. platform. + 4 (C) The direction of frictional force on the block at any instant will be cos (ot) i + sin(wt)} (D) Direciton of friction at any instant will be sin (ot) i+ cos(ot i isc is rotated with . Water is completely filled in a closed disc is rotated an angular velocity © about its symmetry axis in a ity free space. Assume 2270 pressure at . Then eee eae] port? (A) Pressure will vary inside the dise as —— ‘centrifugal forces on any water element must be balanced by field pressure forces. erp ghter particle was inthe fuidit wil havea tendency to get accumulated towards the cee {Dp Ifa heavier panicle was suspended in the liquid, twill havea tendency o get collected towards the centre due to buoyant force. "Sara, Sarwaprya Vihar, Sew Dethi-1 10016, Fh a webste: www fitjee com : 16. Vv. 18, 19, 20. GMP=IS\6-FH-Assion-201 ‘Water flows steadily through a hori rizontal pi : a point where the velocity of low is vat pongo ible cross-section ‘The following statements are given below nt POit (Pressure, wi Oy ext when n<1 oe ) 1 Pp : If the pressure of water is where the velocity of flow in ny © &>4whenn bwhen n> The intensity of light falling on is i second constant, Assur i aa ats a ae ae the no of photons falling per (A) stopping potential will increase, pep te (B) saturation current will increase, (©) saturation current will decrease (D) saturation current will remain unaffected. ‘The position vector of a particle is given by the relation? =G(I-1+Br’), where @ is a constant vector while, B and y are positive constants. Which of the following statement is true? (A) Displacement in first two seconds is &(1-2y +48) (B) Velocity at t= 0 is—ay. (C) Acceleration at t=O is 2Ba. (D) Speed is decreasing with time at t= 0. A block of mass m is pushed towards a movable wedge of mass n Pp and height h, with a velocity u. All surfaces are smooth. Choose the correct statement from the following. (A) Block will reach top of the wedge ifu = Pen(i—+) NS As [] 1 (B) Block will reach top of the wedge if u = ben(is4) (C) Ifthe block overshoots P, the angle of projectile isc. (D) If the block overshoots P, the angle of projectile is Jess than ct. A particle executes simple harmonic motion between x = A and x= +A. The time taken for it to move A Ks. =A tox=& is Te. Then =A is and to move from = tox= ste T,=T: (Ona2 n-th (OT=2% from x = 0 to it immersed in a beaker filled with indrical block of density d stays fully imme ts Fs ea Le liquids of different densities dy ae Te, bed is i cquilibrium with half of it in liquid 1 and the citectal aul are 5 is given small downward displacement, neglecting i jc motion (A) it executes simple harmonic me i ion ii i SHM (B) it motion is oscillatory but not ae juency: jllation depends on size OF PC ies a ©) Se ieee ‘cylinder is symmetric about its equilibrium post GMP-1516-PH-ASSIGN-232 21 22, B3: 24. D5) 26. ” PUTJEE House, 29-A, Kaki Sarai, Sarvapriya Vihar, Neu Deli 1 10010, PuTTe€ Lic, PUTS eee as Eight identical droplets, each falling under gravity in the earth's atmosphere with terminal vetociyy 4 Combine to form a single drop. The terminal velocity of single drop and drag force on it are catentateg Find the correct statement from the following. (A) Final terminal velocity is 4y, (B) Final terminal velocity is 8 V (C) Drag force is increased four times. (D) Drag force is increased eight times, A solid cube of side a and another solid cube of same material and dimension, having, spherical cay of radius (b ah Then, which of the following statement is correct? Vessel A Vessel B (A) If both vessels are stopped suddenly, water in vessel A will be at higher temperature, B) Ifboth vessels are stopped suddenly, water in vessel B will be at higher temperature (©) Change in temperature of water in vessel A is 1.58°C (D) Change in temperature of water in vessel A is 1.3°C. Specific heat of a substance can be (A) finite (© zero (B) infinite (D) negative At ordinary temperatures, the molecules of an ideal gas have only translational and rotational kinetic energies. At higher temperatures, they may also have vibrational energy. As a result, at higher temperatures (A) C, = 3R/2 for monatomic gas (B) C,> 32/2 for monoatomic gas (C) C, < 5R/2 for diatomic gas (D) C,> 5R/2 for diatomic gas A gas perform a process as shown in V/T diagram. Choose the correct option(s), V (A) The process cannot be represented as P"V"T*= constant, where a, b, ¢ are real = numbers. (B) specific heat capacity is dependent on temperature ; (©) Pressure of the gas monotonically decreases (D) pressure of the gas monotonically increases The temperature of an isotropic cubical solid of length L, density d and coefficient of expansion «is raised by 10°C. To a good approximation, at final temperature (A) length is L(1 + 10a) (B) surface area of any face of the cube is L’(1 + 20a) (C) density is dl + 300) (D) density is d/(1+30a) GMp- *SIO-PH-ASSIGN-239 28, From the following statements, ¢o, 29. A revolving ring in a siren revolves 420 ti tuning fork of frequency 348 Hz. Choose th (A) No. of holes in ring = 49. (©) Frequency of siren is 350 Hz. Imes per minute and it produces 2 beats per second with a ie correct statement from the following. (B) No. of holes in ring = 50, (D) Frequency of siren is 342 Hz. we A sinusoidal wave produced in a stretched string gets reflected and y : m — loses 36% of its energy in getting reflected from the fixed end of the string. Choose the correct statement from the (a) Amplitude of reflected wave ie Amplitude of incident wave 10 e (B) Phase difference between incident and reflected wave is x (©) Fieauency of reflected wave _ 4 Frequency of incident wave 5 (D) None of the above. 55. Consider a source of sound S, and an observer P. The source emits sound of frequency No. The frequency observed by P is found to be: N; if P approaches S at speed v and S is stationary, Nz if S approaches P at speed v and P is stationary, and Ns iffeach of P and S have speeds v/2 towards one another. Then, ) (A) Ny =N2= Ns (®) NNo (D) Ns lies between Nj and No 32, A parallel-plate capacitor has plate area A, the distance between its ae caer Aric slab of dielectric constant &, two plates x and y is d and a dielect = is filled between the regions. It is connected to a cell of emf E, If ieee a the dielectric slab is removed, then (A) 24 fe, -1]E charge is absorbed by the cell d (B) 4 fe, ~I]E charge flows through the circuit ae i late x (C) negative charge flows from plate y to pl (D) negative charge flows from plate xto platey i a) ZB. Three identical plates of area A are arranged as shown, A charge +Qis (") a |o given to plate (2). Then, (A) charge +2Q/3 appears on plate 1 (B) charge -2Q/3 appears on plate 1 Ale (©) charge -Q/3 appears on plate 3 () charge on two faces of plate 2 are 2Q/3 and Q/S Hh - ae Papa 10016, Fh 16106000, 2656 ‘Sarvapriya Vihar, Ne 110016, ‘Falt Sarai, suiacon Lid, FUTJEB House, 29-A, OMP=1516-PH-ASSIGN-294 M. As. ah A, B and C are three concentric metallic shells, Shell A is the innermost and shell C is the outermost A is given some charge. Then, (A) Inner surfaces of B and C will have same charge. (B) Inner surfaces of B and C will have same charge density. (C) Outer surfaces of A, B and C will have same charge. () Outer surfaces of A, B and C will have same charge density. When a positively charged sphere is brought near a metallic sphere, it is observed that a force of attraction exists between the two. It means that (A) the metallic sphere is necessarily negatively charged (B) the metallic sphere may be neutral (C) the metallic sphere may be negatively charged (D) none of the above A voltmeter of resistance 600 when connected in tum across = 6000 resistances Ry and Rz gives readings of V; and Va, respectively. If the #1 battery is ideal, then = 3000 (A) Vi =60V (B) V2=30V Ra (© Vi=45v ©) V2=75V For the circuit shown in the adjacent figure, select the correct statements from the following. (A) x and y are equipotential points, “ @) Effective resistance between A and B is 2.0. (©) Effective resistance between A and Bis 1 2. (D) None of the above. A uniform current | is flowing in a long wire of radius R. If the current is uniformly distributed across the ‘cross-sectional area of the wire, then (A) magnetic field increases linearly from centre to surface (B) magnetic field decays inversely with distance r from the centre of wire forr> R (©) magnetic field at the centre of wire is zero (D) None of the above A panicle of charge +q and massim moving under the influence ofa uiform py clectric field Ei and a uniform magnetic field B k, follows trajectory from P 4 x oe to Qas shown in the figure. The velocities at P and Q are Vi and ~ 2V} il i ich of the following statements) is/are true? is ear =. (AyE=5— wv my" . al @) Rate of work done by electric field at Pis 5 (©) Rate of work done by electric field at P is not zero (D) Rate of work done by both the fields at Q is not zero. ee TUBE House, 29-A, Kahu Sarai, Vihar, New Dethi-110016, Fh 41 6 ‘pie Lid, FITIES House, 2A, Savanna Di (05000, 40. 42. SS GMP-1S16-PH_AssiGN-235 A charged particle with velocity = magnetic force on particle is F. Which — fA) No force acts on particle if x = y. AO Force acts along z-axis if x> '+¥J Moves in a magnetic field B = Of the following statements are correct? AB) Fo(x? ~ y) if x>y. (D) Force acts along y-axis if y > x +99. The magnitude of yelocity o such that the angle 0 is he ane © is Constant. Choose the correct statement from the following (A) emf in the loop is cos. (B) emf induced in the loop is zero, (©) emf must be induced as the loop crosses magnetic lines (D) emf must not be induced as flux does not change with time. A uniform circular loop of radius “a” and resistance “R” is placed perpendicular to a uniform magnetic field B. One half of the loop is rotated about the diameter with angular velocity @ as shown. Then, the current in the loop is (A) zero, when 0 is zero (eer Ries (©) zero, when 0 = /2 De pienbaas? AR iy A conducting wire of length ¢ and mass m can slide without friction on two parallel rails and is connected to capacitance C. Whole system lies in a magnetic field B and a constant force F is applied to the rod. Then (A) the rod moves with constant velocity (B) the rod moves with an acceleration of E m+Brfc (©) there is constant charge on the capacitor. (D) charge on the capacitor increases with time In the figure frequency of the A.C. source is 50/x Hz with pon re\ le L= 100 H, C= 1 pF, R= 10 kQ. When all the switches are open V> reads 100 V. All the voltmeters are hot wire voltmeter which read rms, value of voltage. Initially $1, S2 and Ss are open. ® (A) At any instant voltage across R increases at that instant fies magnitude of potential difference across inductor and capacitor simultaneously increases or decreases. (B) Reading of all other voltmeters are also equal to 100 V. (©) When S; ‘and S> both closed reading of V, remains 100 V. (D) When only S; closed reading of Vs decreases. (Wilee Ld, FIJEE House, 2A, Kobe har, New De 10016, Ph 46105000, 20557489, Tuebate: wun fitjon GMP-1516-PH-ASSIGN-236 4. Two thin slabs of refractive indices ji) and j12 are placed parallel to each f other in the x-z plane. If the direction of propagation of a ray in the two media are along the vectors «= «== «aitbj and % =ci+d]j and i is unit vector along the common normal of the slabs, then we have b a a (@) = ® = Vase Vcvd® Vat vc +d’ © ¥-(fixz)=0 (D) §x(axm)=0 J® __ Abi-convex lens is placed between a light source and a concave mirror as ‘shown such that image of the light source coincides with itself. Then, (A) light, after being refracted, may fall normally on the mirror ee (B) light, after being refracted, may fall at the pole of the mirror (© source and its image can coincide for two positions of source ©) none of the above ST. A particle moves towards a concave mirror of focal length 30 cm along its axis and with a constant speed of 4 cm/sec. At the instant the particle is 90 om from the pole — (A) velocity of image is 1 cm/sec — B) velocity of image w.r.. particle is 5 cm/sec © particle and image move towards each other (D) as the particle approaches pole, velocity of image increases A Young’s double slit experiment is conducted in water (1) as shown in the figure, and a glass plate of thickness t and refractive ater index p12 is placed in the path of Sz. Then, en i be (A) Optical path difference at ‘O” is (ea): : ce Hy Is (B) Optical path difference at “O” is (2 — p)t. (C) Optical path difference is independent of position of glass plate, provided light from Sz passes through it. (D) None of the above 45, A solid transparent sphere made of sans =n> Dow a small, opaque dot at its centre, When observed from outside, apparent position (A) closer to eye than its actual position (B) farther from eye than its actual position (C) same as its actual position (D) independent of refractive index of the sphere 52a A Simincts point object is placed at O, whose image is formed at I as 01 shown in the figure. AB is the optic axis. Which of the following statements are correct? - : a (A) Ifa lens is used to obtain image, the lens must be converging. ere ioe é i (B) Ifa mirror is used to obtain image, the mirror must be a cor si a the point of intersection of lines OI and AB, vex mirror having pole at the point (C) Position of principal focus of mirror cannot be found. (D) Lis real image. ST TTIBE House, 29-A, Kalu. Sarai, Sarvapriya Vihar, New Delhi 1100 rare Ld, ue Dei -110016, P4610 51 53. OO —— GMP-1516-PH-ASSIGN-237 In Fresnel bi-prism experiment a very greater than of prisn m. Angl is slightly vertically upward then Ble 0 is ve When photons of energy (he/,) fall on a function of the surface is hv,, then (A) maximum kinetic energy of the electron is (® ~hy, ) at metal surface, photoelectrons are ejected from it, If the work (B) maximum kinetic energy of the photoelectron is equal to (ha/.) (C) minimum KE, of photoelectron is zero, (D) minimum kinetic energy of the photoelectron is equal to he/A. ‘The threshold wavelength for ‘be emitted when the material i (A) 50 watt infrared lamp (©) 50 watt ultraviolet lamp Photoelectric emission from a material is $200 A. A photoelectron will s illuminated with monochromatic radiation from a B) | watt infrared lamp (©) | watt ultraviolet lamp Photoelectric effect supports the quantum nature of light because: (A) There is a minimum frequency of light below which no photoelectrons are emitted. (B) The maximum KE. of photoelectrons depends only on the frequency of light and not on its intensity. (© Even when the metal surface is faintly illuminated by light of wavelength less than the threshold wavelength, the photoelectrons leave the surface immediately. (D) Electric charge of photoelectrons is quantized. When photoelectrons of energy 4.25 eV strike the surface of metal A, the ejected photoelectrons have maximum kinetic energy Ta eV and de-Broglie wavelength 2. The maximum kinetic energy of photoelectrons liberated from another metal B by photons of energy 4.70 eV is Ty = (Ta ~ 1.5) eV. If de-Broglie wavelength of these photoelectrons is Ap = 22a, then (A) work function of A is 2.25 eV. (B) work function of B is 4.20 eV (© Ta=2.00 eV (D) Ty=2.75eV ‘The potential difference applied to an X-ray tube is increased. Which of the following statement is true about the radiation emitted? (A) Intensity of the radiation increases (B) Minimum wavelength of the radiation increases (©) Intensity of the radiation remains unchanged. (D) Minimum wavelength of the radiation decreases. ‘Threshold wavelength of certain metal is 4,. Light of wavelength slightly less than 2, is incident on the plate, Then, choose the correct statement from the following iti ill come out from the plate. . ® te eds ee retarding force due to development of positive charges on the plate. (C) After some time, ejection of electrons stop. (D) None of the above Srapriya Vinar, New Delhi -1 10016, Ph 46106000, 26508 ‘wubotte: wi flee com. FMUGC Lid, PUTIEE House, 29-A, Kalu Sava, Sar Oe ar 53. GMP-1816-PH-ASSIGN-237 ©) dist aie (D) None of the above by biprism remain unchanged, 4 When photons of energy (he/A) fall on a meta rons are ejected from it. If the work ictal surf ee ; ; face, photoelectrons are ejected fi ro (A) maximum kinetic energy of the electron is ( (B) maximum kinetic energy of the photoelectron is equal to (he/A) (©) minimum KE, of photoelectron is zero, (D) minimum kinetic energy of the photoelectron is ‘equal to he/. ‘The threshold wavelength for photoelectric emission from a material is 5200 A. A Photoelectron will be emitted when the material is illuminated with monochromatic radiation from a (A) 50 watt infrared lamp (B) 1 watt infrared lamp (©) 50 watt ultraviolet lamp (D) 1 watt ultraviolet lamp Photoelectric effect supports the quantum nature of light because: (A) There is a minimum frequency of light below which no photoelectrons are emitted. (B) The maximum K-E, of photoelectrons depends only on the frequency of light and not on its intensity. (©) Even when the metal surface is faintly illuminated by light of wavelength less than the threshold wavelength, the photoelectrons leave the surface immediately. (D) Electric charge of photoelectrons is quantized. When photoelectrons of energy 4.25 eV strike the surface of metal A, the ejected photoelectrons have maximum kinetic energy T, eV and de-Broglie wavelength 2. The maximum kinetic energy of photoclectrons liberated from another metal B by photons of energy 4.70 eV is Ty = (T, ~ 1.5) eV. If de-Broglic wavelength of these photoelectrons is A= 2A, then ; (A) work function of A is 2.25 eV (B) work function of B is 4.20 eV (©) Tr =2.00 eV @) Ta=2.75 eV ‘The potential difference applied to an X-ray tube is increased. Which of the following statement is true about the radiation emitted? (A) Intensity of the radiation increases (B) Minimum wavelength of the radiation increases (C) Intensity of the radiation remains unchanged. (D) Minimum wavelength of the radiation decreases, ‘Threshold wavelength of certain metal is 2,. Light of wavelength slightly less than 2. is incident on the plate. Then, choose the correct statement ia Be following ‘A) Initially, electrons will come out from the plate. s ® The daa ios experience retarding force due to development of positive charges on the plate, (C) Afier some time, ejection of electrons stop. (D) None of the above GMP-1516-PH-ASSIGN-238 SA gas took part in three thermal processes in which it is heated from the Same initial state to the same final temperature, The process are shown on the P-V diagram by straight lines 1-2, 1-3 and 1-4. Q,, Qz and Qs are mount of heat supplied to the gas in the process 1 -> 2, 1 > 3, 1 > 4. respectively. AW>Q ) Qi < Qe OQ DO O1> 59. A A frame a bed and a sliding rod PQ of resistance R, start moving with the velocities v and 2v respectively parallel to a long wire carrying steady current I, as shown in the figure. (A) charge on the capacitor at time tis q = Sain? eens] i B) charge on the capacitor at time tis q= ere i 6°) 7 5 ivin2, cc (© current passing through the resistor at ime tisi= "SAF o-v"e] i inc} Two spheres A and B have the same radii but the capacity of A is greater than that of B. The surfaces of both are painted black. They are heated to the same temperature and allowed to cool in vacuum. ‘Then, (A) A cools faster than B (B) both A and B cool at the same rate (©) at any temperature the ratio of their rates of cooling is a constant (D) B cools faster than A an angle 9“with an initial speed u. ‘momentum about the point of projection is Mg u cos 6 t. pyivin2 aR (D) current passing through the capacitor at time tis i= A particle is project (A) its rate of change of (B) its angular mome! (©) it will cover equal distances in eqial intervals of time, (D) the radius of curvature ofits trajectory will be maximum at the highest point. : point of projection at any time is Mgu cos 0 5 ‘A-canon of mass M is mounted on an east west frictionless railway track. The barrel of cannon faces the north east direction at an angle of ‘45° with the vertical. The canon fires a shot of mass M with a speed V relative to the barrel. ‘ (A) The recoil speed of the cannon will be V/4 VB y “eee ) The speed ofthe shot as seen from the ground willbe ~~ V. _— (C) The momentum of the shot + canon system will remain conserved along the east west line. (D) The momentum of the shot plus canon system will remain conserved along the North South line. ‘A particle is launched from a height 8R above the surface of earth and itis given a speed on parallel to the surface then {A) It-will escape fro the gravitational field of earth, (B) it will follow a circular orbit, (©) twill follow an elliptical orbit, {D) the point of launching will be the perigee ofits orbit. GMP-1816-PH-ASSIGN-239 64. Arod AB of mass M and ten, imparting the impulse: (A) The radius of curvature of trajectory of A as seen from the ground is ®1 (B) The radius of curvature of trajectory of B as seen from the ground is 2, 8 (©) The instantaneous axis of rotation is ata distance of L/6 from the mid point of the rod. (D) the mid point of the rod will move along a straight line 65. A rod of mass m and length £ suspended from a point swings in a vertica plane such that magnitude of acceleration of its C.M. at extreme points is same as that at the mean point then: (A) The deflectin angle from the vertical to the extreme position is cos(3/5) (B) At the extreme position there is no centripetal acceleration of the CM. (©) At the mean position there is no tangential acceleration of the CM. (D) The angular momentum of the rod about the suspension point remains constant. Read the paragraph carefully and answer the following questions: A regular hexagon of side a = 10V3 m is kept at horizontal surface shown in the figure, A particle is projected with velocity V fe fat an angle 0 from point K such that it will just touches the all a comers regular hexagon at B, C, D and E as shown in the pee O is the centre of hexagon and x-axis is parallel to horizontal : y-axis is perpendicular to AF and CD sides of hexagon then answe the following {take g = 10 m/s") 1 ‘The maximum height from horizontal surface attained iy paste is 10m ebm 2. “The velocity of projectile at maximum height ae, (A) 553 m/s ee (© 10%3 m/s i i cle from D to Bis 3 ‘The time required to move the particle from ae oO 5 (D) 2.5 sec see ee ypc, Hew Debi-110016, PR 46105000, 26569489, RUNGE Lid, FCTIEE House, 294, Kat Sarai, ya ba GMP-1516-PH-ASSIGN-240 Read the paragraph carefully and answer the following questions: A rod AB of length 2m moves in horizontal x-y plane. At any instant end a of pe the rod is at origin and has velocity V, = 2 +v,j. The other end B at the same”) 4 4% instant is moving with velocity V, =3i+6]. The rod makes an angle of 30° 30" with the x-axis at this instant (see figure.) {fe ¥pnzteyl 4. Read the paragraph carefully and answer the following questions: A Cs plate is irradiated with light of wavelength 1. = aa being = the work function of the plate, h plank constant and c velocity of ||S—> light in vacuum. Assume all the photoelectrons are moving perpendicular to the plate towards a YDSE setup when accelerated ¢, through a potential difference V. Take charge on an electron = ¢ and The magnitude of angular velocity of the rod is. (A) 1 rad/sec @®) V3 rad/sec © BS rad/sec (D) 12 rad/see The value of vy is (A) 6 m/s @) + V3) ms © 6V3) ms () none of these The component of velocity of centre of mass in the direction perpendicular to the rod at the given instant is @ er @) zero (© w3 -4 (D) None of these mass of an electron = m . as ‘The fringe width due to electron beam is w 2 @ 2 bD (O reag (D) none of these If the wavelength of light used in photoemission is less than inge width wil (A) increase @) pee ee (C) remain same GMP-1516-PH-ASSIGN-241, Instead of moving perpendicular t 9 maximum would shift © the plate, ifthe electrons were movi (A) upward ing randomly, then the central (C) no shift (B) downward (D) no fringes will be formed Read the paragraph carefully and answer the following questions: Many interesting wave phenomenon in nature cannot just be deser i anlyae complex waveforms in ems of & combinations of ay wire Ta combinations, we make use of the principle of superposition which states that if two ¢ travellis are moving through a medium and combine at a given point, the resultant dips Git dit iat point is sum of the displacement of individual waves. Two pulses travelling on the same string are described by. 5 Yi epee = 2 (3x-4t) +2 (8x+4t-6) +2 instead one must analyze such wave 10, The direction in which each pulse is travelling (A) yi is in positive x-axis, y2 is in positive x-axis, (B) y; is in negative x-axis, yo is in negative x-axis. (© Ji is in positive x-axis, y2 is in negative x-axis. (D) y1 is in negative x-axis, yo is in positive x-axis. 11, The time when the two waves cancel everywhere (A) 1 sec (B) 0.5 sec (© 0.25 sec (D) 0.75 seo 12, The point where the two waves always cancel (A) 0.25 m )0.5m (© 075m the paragraph carefully and answer the following questions: Arod of mass m and length £ is rotating about a fixed point in the ceiling with an_ angular velocity o as shown in the figure. The rod maintains a constant angle © with the vertical, 13. What will be the horizontal component of angular momentum of the rod about the point of suspension in terms of m, @, £and 0, mo] mol (Ay cos @ ME sino 2 2 mol mol’ |. z mol’ sind SE sin20 os sntum of the rod? M4. Whats the rate of change of angular momei ee (A). mo’ f* sin® ) a GMP-1816-PH-ASSIGN-242 15. What " ry the rod make with the vertical (A) cos" 32, (B) cos" (3; a aare ©on'(s28) () eee COMPREHENSION = VI eee Paragraph carefully and answer the following questions: BESS S ass m, can slide freely on a frictionless, horizontal “¢ and ie B are attached to an inextensible, inelastic cord of length oe Are at rest in the position shown where sphere B is struck by sphere C os 5S moving to the right with a velocity va. Knowing thatthe cod is taut Tere sphere B is struck by sphere C and assuming “Head on” inelastic impact en B and C, we can’t conserve kinetic energy of entire system. 16. The velocity of B immediately after collision is along unit vector wi i OR (D) Can't predict 17. Velocity of A immediately after collision is along unit vector : 1; (Aji = yi ® Fie ©; (@) Can't predict 18. If velocity of C immediately after collision becomes = in the initial direction of motion, the impulse due to string on sphere A is ve a og Ole gum (©) none of the above 2 Read the paragraph carefully and answer the following questions: ‘A particle of mass 1 kg is projected at an angle of O = w/4 from horizontal with a muzzle velocity of 20 mls, A long slender rod of tase 3 kg and length 30 m is suspended vertically from a point at the same horizontal level as that of point of projection and at a distance of 60 m from the projection point. The rod can rotate freely, Ifcollision occurs, it is perfectly plastic. (g = 10 wis’) 19. Angular velocity of rod after collision is 1 4 () praise ®) Fr ndisee (C) 4V2 rad/seo () zero ‘Sarvapriya Vihar, New Delhi -1 10016, Ph 46106000, ‘website: wi flljee.com. TUE House, 2°A, Kal Sara, GMP-1516-PH-ASSIGN-243, (B) 6 = cos (40/42) ) () 6 = cos(27/28) 2 if Srey is fired horizontally, it will (A) hit the rod betwee: A i ase nO and A (except mid point) (B) hit the rod at its mid-point (D) not hit the rod at all Read the paragraph carefully and answer the following questions: 4 identical balls of radius R and mass m are lying space. ig in a gravity free space. The balls are in contact and there centres are forming vertices Fes of side 2R in horizontal plane. One identical ball travelling vertically with a speed v hits the four balls symmetrically. The collision is perfectly elastic. The centre of the 4 stationary ball are at (R, R, 0), 4 4 -R, -I os ), (R, -R, 0), (R, -R, 0) and CR, -R, 0) 2. ‘The speed of ball | after collision is: wy By @- 2 a 6 2 © ay3 (D) none of these 2. The speed of ball which was intially travelling with speed v just after collision is: (A) ¥ @ 2% (A) 3 "1 (Ov )2v 24, Mark the incorrect Roget ‘A) Momentum of system will remain cor é ® Fanctic energy of system will remain conserved at all instant. Mechanical energy will remain cor . Coe ” ‘Angular momentum of system about Centre of mass will remains conserved. paragraph carefi ‘and answer the following questions: ; Read the paragraph core of masses and ma Which fe come Y PEN Ce ee arap over alia moon LEY me TT ee aa incase Sn Pa ie ayer an sion ofthe Sing A xP low under different situations: (@ when m, > m,, In this case tT 7 _(™=m pa eee : (Ese (ie (ii) when m, > m,. In this case _(m,-m, Ae ia o-(%=t sas -( 20 When m, = mp =m. In this case a= 0,7 = 8 GMP-1516-PH-ASSIGN-244 25, © 6 kg and g= 10 ns? then what is the acceleration ‘of masses? (B) Sm (D) 40 ns 26, Whatis the tension i sion inthe string? (hati i the string: @BN (© 100N ce 27. IP the pulley is pulled uprvard with acceleration equal (othe acceleration due t0 Bravity, what will be the tension in the string? (A) 75N BUN (©) 300N oer “COMPRENENSION -X Read the paragraph carefully and answer the following questions: A long slender rod of mass 2kg and length 4 m is placed on a smooth ny horizontal table, Two particles of masses 2 kg and 1 kg strike the rod fo" Simultaneously and stick to the rod after collision as shown. am oe ater 28. Velocity of centre of mass of the rod after collision is ts (A) 3 mvs (B) 6 ws (©) 9 mvs O ums —J, hte 29, Angular velocity of the rod after collision is (A) 30/17 rad/sce (B) 19/17 rad/sec (© V3 madisec (D) 30/41 rad/sec 30. If the two particles strike the rod in opposite direction, then afier collision, as compared to the previous situation, the rod will (A) rotate faster and translate slower (B) rotate slower and translate faster (C) show no change in linear or angular velocity __(D) rotate faster but translate at the same rate Read the paragraph carefully and answer the following questions: ‘A 60 mm radius wheel is connected to a fixed support D by two links AB and BD. At the instant shown, the velocity of the centre ‘A of the wheel is 30 m/s to the left. Rod AB and BD have equal length = 5m. 31. The angular velocity of rod AB will be (A) 3 V2 radssec Gina (©) 72 rad/see O):Neceiee 32, ‘The angular velocity of rod BD is (A) 2N2 rad/see ®) 3V2 rad/sec (© 8v2 raise ©) None of these. GMP-ASI-PH-ASSION-245 33, Velocity of pin B will be (A) 162 mis V2 ae: (B) 52 mms (D) None of these, COMPREMENSION XI Read the paragraph carefully and answer the following question: ist When a particle is projected at some angle with the process the horizontal velocity remains constant during flight the acceleration of the p. point the path of particle can be cons curvature of the path. horizontal, the path of the particle is parabolic. In the ae but the magnitude of vertical velocity changes. At any instant c remains g in vertically downward direction. During flight at any sidered as a part of circle and radius of that circle is called the radius of Consider that value of g = particle is projected with velocity u = 10 m/s at an anj i ti Je 0 = 60? with a O m/s"; Now answer the following questions, . te cee 34, The radius of curvature of path of particle at the instant when the velocity vector of the particle becomes perpendicular to initial velocity vector is 20 ) Sem iW 33 ® oF 40 a0 © —.m i 3V3 an” 35, The magnitude of acceleration of particle at that instant is (A) 10 mv/s* (B) 5V3m/s? (©) Sms @) 10/3 m/s? 36. Tangential acceleration of particle at that instant is (A) 10 mvs* (®B) 20 m/s (©) Sms? (©) 5¥3m/s* Read he paragraph carefully and answer thefollowing questions: Ina gravity free space, a hollow cylinder of radius 4R and ey is = sect by a constant angular velocity « clockwise wrt an inet) Tam oe rotated by & her hollow cylinder of radius R and mass m i rating ist™ ‘linder, such that it touches the inner surface of larger cylinder without slipp anne eee of mass appears stationary from ioarial ams DY tie ae eriaea besweda cylinders ia Vel nom tein SS denoted by N, answer the following questions. i linder. 37. Whatis the angular velocity of smaller cy a (A) 03 (D) 30/4 (C)40 38, What is the value of N. (B) mR) (40 (A) m@R) 07 a (D) zero (ym an (30) Thar New bedi -110016, ‘Sarvapriya eto SST Tan PITIEE House, 270A, Kaks Sardi Samet yas fies GMP-1516-PH-ASSIGN-245 a A small tangential velocity v is given to the smaller cylinder towards right. What will be N. Once the Steady state is achieved. (A) m_ 16R © mER Coley" ‘Read the paragraph carefully and answer the following questions: A spherical ball of mass M moving with inital velocity collides elastically with g_54 “ thence ball of mass M which isa of L shaped rigid massless frame as shown in gure. The L shaped frame contains another mass M connected at other end. t le figia) fig(b) 40, The speed of striking mass after collision is (A) w/7 backwards (B) w3 is same direction Oo (D) w2 backwards 41 ‘The angular speed of L frame immediately after collision is u 4u (A) — = ie 7L ®) 2L oF o* 42. How soon will the frame come to the orientation shown in the figure (b) after collision? TxL iL ea or Trl aL le o* eee SION XV 95 Read the paragraph carefully and answer the following questions: Figure shows 4 identical masses of mass m, arranged on a cube as shown, 43. ‘The potential energy of the system is re) 2oSmr (B) 3y2 = 2 (0-22 (0) -32 Sot 44, The force on any particle is 3 GMP ea (D) none of these hin 1001e Boa cei Vihar Mes Debi -110016, he 4670 GMP-1516-PH-ASsIGN-247 45, Ione of the masses is released (A) It will collide with one of the masses (©) It executes SHM (B) It execute an oscillatory motion ( (D) none of these eee onieneNsioMe x Read the paragraph carefully and answer the following questi ions: The word fluid means a substance having parti a icles wh may appear to be of negligible will cause deformation nt density. Specific weight, specific gravity, volume and it is denoted by. The specific Teadily of its magnitude (a small shear st cae in the fluid), Fluids are charactrised by such aces Mtn Y ce. Density of a substance is defined as mass per unit commonly taken as a reference subst: “ty Tepresents a numerical ratio of two densities, and water is stance. Specific gravity of a substance in written as the ratio of d Be is Wael es water. Specific weight represen the force exerted by gravity on a unit volume ot luid. It is rela he density as the product of density of a fluid and accelerati the most important and is recognized asthe only single propery which influences the fad moon to ct extent, The viscosity is the property by virtue of which a fluid offers resistance to deformation under the influence if shear force. The force between the layers opposing relative motion between them are known as forces of viscosity. When a boat moves slowly on the river remains at rest. Velocities of different layers are different. Let v be the velocity of the level at a distance y from the bed and V + dv be the velocity at a distance y + dy. The velocity differs by dy in going through a distance by perpendicular to it. The quantity & is called ly velocity gradient, The force of viscosity between two layers of a fluid is proportional to velocity gradient and ‘Area of the layer. dy «eA& Fa — F Cay F=-nA x (ve sign shows the force is frictional in nature and opposes relative motion. 7 ly coefficient of dynamic viscosity Feed Shear stress © = eK aay and simultaneously kinematic viscosity is defi as, The viscosity ofa fluid depends upon is interme ed resulting in a negligible intermolecular cohesion, Sa ane seh larger with te increases of temperature, the cohesve force decreases rapidly resulling in the decreases of viscosity. Incase of gases, the viscosity is mainly du 6 transfer of molecular ‘momentum in the transverse direction brought about by the molecular agitation. Molecular agitation incase aera ae enare, Thos we conclude that viscosity ofa id may ts be conse 1 composed o two pants fist that due fo intermolecular cohesion and second due 0 tanser of molecular momentum. c gravity 1.3 is mixed wit 6 lites of liquid of specific gravity 08 the ined as the dynamic viscosity divided by the density. If is denoted :ntermolecular structure. In gases, the molecules are widely while in liquids the molecules being very close to each 46. Ten litres of liquid of specifi specific gravity of mixture 1s @® i w ae (D) none of these 47. If the velocity profile is given by a the bed. Determine shear stress at y = 0.15 m, & v= 2y-ytv is velocity in m/sec y is in meter above 1 = 0.863 Ns/m? els (A) 0.316 wile (©) 0.50 a iar New Dat 110016, Ph 46109000, 7656048 Se STEE House, 29-A. Kaks Sarai, Sap ny fitj.com. GMP-1516-PH_ASSIGN-248 48. Viscosity of liquids (A) decreases with increase in fluid temperature (B) increases with increases in fluid temperature (© in only due to transfer of molecular momentum of the molecule (D) is dependent of pressure COMPREHENSION -XVIL : Read the paragraph carefully and answer the following questions: ‘When we raise the temperature of a body, the molecules and atoms move with Greater kinetic energy. Assume that “the temperature” is the manifestation of only the Kinetic energy in atomic and subatomic levels and no mass change (exces/defect) occurs due to heating. Now a solid material is supplied with heat al a constant rate, ‘The temperature of the material is changing with the heat input as shown in the graph. BC and DE are parallel to the Heat-input axis 49. In BC and DE (A) kinetic energy increases (B) potential energy increases (C) kinetic energy decreases (D) potential energy remains constant 50, IN AB, CD and EF, the energy of the system increases. Here “energy” refers to (A) kinetic energy (B) potential energy (C) both kinetic and potential energy (D) none of these 51. __Inwhich of the following phase the assumption made in the above comprehension is most precise? (A) solid ©) liquid (©) gas (D) not predictable Read the paragraph carefully and answer the following questions: ‘A beaker containing an ideal fuid executes plane SHM in a horizontal plane according to the equation x = 28 sn, being the mean position. A bob is suspended at S through a string of length L as shown in the figure. The line SO is vertical. Assuming L >> & ‘The tension in the string is maximum at time t= $2. (A) Wo ®) wo (© wo (D) none of these 53, The magnitude of maximum buoyant force and the time with it occurs for the second time ar, respectively (A) 2g, x20 ®) £30 (© 2g, 3x2 ©) g,Wo ie TOA Kobe nary ers Tid Td. PTIEE ose, 7A as Sart, Supra Vina New DRT T TOOT GOTO OMP-ASVE-FH-A88I0M 249 54, The direction of buos “w vant force at time t= © : 1/20 is best represent ) ae, Coenen Read the paragraph carefully and answer the following questions: ‘Two coherent point sources of sound wave S; and Sz produce sound of same frequency 50 Hz and wavelength 2 cm with amplitude 2 x 10? m. Each circular aro represents a wavefront at a particular time and is separated from next arc by a distance 1 om. Both the sound waves propagate through the medium and interfere with each other. Read paragraph carefully and answer the following questions. [r= 1 cm] 55, The point(s) where constructive interference occurs (A) G only ®) Pand A (© GandF () TandU 56, The points of minimum intensity will occur at (A) FandR (B) GandP (©) GandF (D) Fand P 57. How many of the points shown in the figure represent maxima (A) 10 @) ue © 13 @) Read the paragraph carefully and answer the following questions: ivencharges g{ gy 9 7 hh of area A of a parallel plate capacitor are given < 3 : Oaueate ar are joined by a non-conducting spring of force = oa TF The natural length of the spring is d the intial SepAON sciueatle — d= between the plates. The left plate is connected t0 a vertick eh 2 massless non-conducting rope and the right plate caus diag block of mass m through similar Tope. oe massless, neglect dielectric effect of the spring mACoE. i ‘maximum elongation of the spring is 58. Ie the block is reeased om see “ (a) i) 2(me--2-) KU os { g (D) None of these.

You might also like